0% found this document useful (0 votes)
17 views92 pages

Quantity App

The document provides an overview of the concepts of Highest Common Factor (HCF) and Lowest Common Multiple (LCM), including their definitions, properties, and methods for calculation such as prime factorization and division methods. It includes examples to illustrate how to find HCF and LCM for various sets of numbers, as well as formulas relating HCF and LCM. Additionally, it discusses the significance of these concepts in mathematics and provides problem-solving examples to reinforce understanding.

Uploaded by

laita nikam
Copyright
© © All Rights Reserved
We take content rights seriously. If you suspect this is your content, claim it here.
Available Formats
Download as PDF, TXT or read online on Scribd
0% found this document useful (0 votes)
17 views92 pages

Quantity App

The document provides an overview of the concepts of Highest Common Factor (HCF) and Lowest Common Multiple (LCM), including their definitions, properties, and methods for calculation such as prime factorization and division methods. It includes examples to illustrate how to find HCF and LCM for various sets of numbers, as well as formulas relating HCF and LCM. Additionally, it discusses the significance of these concepts in mathematics and provides problem-solving examples to reinforce understanding.

Uploaded by

laita nikam
Copyright
© © All Rights Reserved
We take content rights seriously. If you suspect this is your content, claim it here.
Available Formats
Download as PDF, TXT or read online on Scribd
You are on page 1/ 92

Unit 1 Number system –Basic

1)HCM AND LCF


Properties of HCF and LCM: For
the better understanding of the concept LCM (Lowest
Common Multiple) and HCF (Highest Common Factor), we need to recollect the
terms multiples and factors. Let’s learn about LCM, HCF, and relation
between HCF and LCM of natural numbers.

Definition of LCM and HCF


Lowest Common Multiple (LCM): The least or smallest common multiple of any two or
more given natural numbers are termed as LCM. For example, LCM of 10, 15, and 20
is 60.
Highest Common Factor (HCF): The largest or greatest factor common to any two or
more given natural numbers is termed as HCF of given numbers. Also known as GCD
(Greatest Common Divisor). For example, HCF of 4, 6 and 8 is 2.
4 = 2 × 2
6 =3 × 2
8 = 4 × 2
Here, the highest common factor of 4, 6 and 8 is 2.
Both HCF and LCM of given numbers can be found by using two methods; they are
division method and prime factorization.

HCF and LCM Formulas


Property 1: The product of LCM and HCF of any two given natural numbers is
equivalent to the product of the given numbers.
LCM × HCF = Product of the Numbers
Suppose A and B are two numbers, then.
LCM (A & B) × HCF (A & B) = A × B
Property 2: HCF of co-prime numbers is 1. Therefore LCM of given co-prime numbers
is equal to the product of the numbers.
LCM of Co-prime Numbers = Product Of The Numbers
Property 3: H.C.F. and L.C.M. of Fractions
LCM of fractions = LCMofnumerators

HCFofdenominators

HCF of fractions = HCFofnumerators

LCMofdenominators

HCF
The greatest number which divides each of the two or more numbers is called HCF or
Highest Common Factor. It is also called the Greatest Common
Measure(GCM) and Greatest Common Divisor(GCD). HCM and LCM are two different
methods, whereas LCM or Least Common Multiple is used to find the smallest common
multiple of any two or more numbers.
Example: The Highest common factor of 60 and 75 is 15 because 15 is the largest
number which can divide both 60 and 75 exactly.
We can find the HCF of any given numbers by using two methods:

• by prime factorization method

• by division method

HCF By Prime Factorization Method


Follow the below-given steps to find the hcf of numbers using prime factorization
method.
Step 1: Write each number as a product of its prime factors. This method is called
here prime factorization.
Step 2: Now list the common factors of both the numbers
Step 3: The product of all common prime factors is the HCF( use the lower power of
each common factor)
Let us understand with the help of examples.
Example 1: Evaluate the HCF of 60 and 75.
Solution:

Write each number as a product of its prime factors.

22 x 3 x 5 = 60
3 x 52 = 75
The product of all common prime factors is the HCF( use the
lowes power of each common factor)
The common prime factors in this example are 3 & 5.
The lowest power of 3 is 3 and 5 is 5.
So, HCF = 3 x 5 = 15
Example 2: Find the HCF of 36, 24 and 12.
Solution:

Write each number as a product of its prime factors.

22 x 32 = 36
23 x 3 = 24
22 x 3 = 12
The product of all common prime factors is the HCF( use the
lowes power of each common factor)
The common prime factors in this example are 2 & 3.
The lowest power of 2 is 22 and 3 is 3.
So, HCF = 22 x 3 = 12
Example 3: Find the HCF of 36, 27 and 80.
Solution:
Write each number as a product of its prime factors.

22 x 32 = 36
33 = 27
24 x 5 = 80
The product of all common prime factors is the HCF( use the
lowes power of each common factor)
The common prime factors in this example are none.
So, HCF is 1.

HCF By Division Method


You have understood by now the method of finding the highest common factor using
prime factorization. Now let us learn here to find HCF using division method.
Basically division method is nothing but dividing the given numbers simultaneously
to get the common factors between them. Follow the steps mentioned below to solve
problems of hcf.

• Step 1: Write the given numbers horizontally, in a sequence, by separating it


with commas.
• Step 2: Find the smallest prime number which can divide the given number. It
should exactly divide the given numbers. (Write in the left side).
• Step 3: Now write the quotients.
• Step 4: Repeat the process, until you reach the stage, where there is no
coprime number left.
• Step 5: We will get the common prime factors as the factors in the left-hand
side divides all the numbers exactly. The product of these common prime
factors is the HCF of the given numbers.
Let us understand the above mentioned steps to find the hcf by division method
with the help of examples.
Problem 1: Evaluate the HCF of 60 and 75

As we can note that the mentioned prime factors in the left side divide all the
numbers exactly and so, they all are common prime factors. We have no common prime
factor for the numbers remained at the bottom.
So, HCF = 3 × 5 = 15.
Example 2: Find out HCF of 36, 24 and 12

HCF = 2 × 2 × 3 = 12.
Example 3: Find out HCF of 36, 24 and 48.

HCF = 2 × 2 × 3 = 12.

HCF by Shortcut method


Steps to find the HCF of any given numbers.

• Step 1: Divide larger number by smaller number first, such as;


Larger Number/Smaller Number

• Step 2: Divide the divisor of step 1 by the remainder left.


Divisor of step 1/Remainder

• Step 3: Again divide the divisor of step 2 by the remainder.


Divisor of step 2/Remainder

• Step 4: Repeat the process until the remainder is zero.


• Step 5: The divisor of the last step is the HCF.

How to find the HCF of 3 numbers

1) Calculate the HCF of 2 numbers.

2) Then Find the HCF of 3rd number and the HCF found in step 1.

3) The HCF you got in step 2 will be the HCF of the 3 numbers.

The above steps can also be used to find the HCF of more than 3 numbers.

HCF Examples
Here are a few more example to find the highest common factors.
Example 1: Find out HCF of 60 and 75.

So, the HCF of 30 and 45 is 15.


Example 2: Find out HCF of 12 and 36.
So, HCF of 12 and 36 = 12
Example 3: Find out HCF of 9, 27, and 48
Take any two numbers and find out their HCF first. Say, let’s find out HCF of 9
and 27 initially.

So, HCF of 9 and 27 = 9


HCF of 9 ,27, 30
= HCF of [(HCF of 9, 27) and 30
= HCF of [9 and 30]

Hence, HCF of 9 ,27, 30 = 3


Example 4: Find out HCF of 5 and 7

Hence HCF of 5 and 7 = 1


LCM - Least common multiple
The definition of LCM states- Least common multiple is a method to find the
smallest common multiple between any two or more numbers. Basically, the common
multiple is a number which is a multiple of two or more numbers. L.C.M is used to
determine the least common factor or multiple of any two or more given integers.
For example, L.C.M of 16 and 20 will be 2 x 2 x 2 x 2 x 5 = 80, where 80 is the
smallest common multiple for numbers 16 and 20. Now, if we consider the multiples
of 16 (16,32,48,64,80,..) and 20 (20,40,60,80,..) we can see that the first common
multiple for both the numbers is 80. This proves the method of LCM as correct.
Along with the least common multiple, you must have heard about the highest common
factor, H.C.F., which is used to derive the highest common multiple factors of any
two or more given integers. It is also called as Greatest Common Divisor. For
example, the H.C.F. of 2,6,8 is 2, because all the three numbers can be divided
with the highest number 2 commonly. H.C.F. and L.C.M. and both have equal
importance in Maths

How to find LCM?


As we have already discussed, the least common multiple is the smallest common
multiple for any two or more given numbers.
A multiple of a number we get, when we multiply a number with another number. Like
4 is a multiple of 2, as we multiply 2 with 2, we get 4. Similarly, in the case of
maths table, you can see the multiples of a number when we multiply them from
1,2,3,4,5,6 and so on but not with zero. Now, if we have to find the common
multiple of two or more numbers, then we have to write all the multiples for the
given numbers. Say for example, if there are two numbers 4 and 6, then how to find
the common multiple between them?
Let us write multiples of 4 and 6 first,
4 : 4,8,12,16,20,24,28,…..
6: 6,12,18,24,30,36,42…..
From the above two expressions you can see, 4 and 6 have common multiples as 12
and 24. They may have more common multiple if we go beyond. Now, the smallest or
least common multiple for 4 and 6 you can see here is 12. Therefore, 12 is the
L.C.M of 4 and 6. Also, find to learn LCM of two numbers here.
Now, let us take an example of 3 numbers.
Example: Find the L.C.M 4,6 and 12.
Solution: First write the common multiples of all the three numbers.
4 : 4,8,12,16,20,24,28,…..
6: 6,12,18,24,30,36,42…..
12: 12,24,36,48,60,72,….
From the above-given multiples of 4, 6 and 12, you can see, 12 is the smallest
common multiple.
Therefore, L.C.M. of 4, 6 and 12 is 12.

LCM Formula
Let a and b are two given integers. We can write the formula for L.C.M. on the
basis of the greatest common divisor(gcd) as mentioned below.
L.C.M. (a,b) = a∗bgcd(a,b)
This is the formula for two integers. But for fractions, the formula of L.C.M.
becomes;
L.C.M. = L.C.MOfNumeratorL.C.MOfDenominator

LCM Example
Example: Find L.C.M. of 10 and 20.
Solution: We know, for given two integers a and b,
L.C.M. (a,b) = a∗bgcd(a,b)
Therefore, L.C.M. (10,20) = 10∗20gcd(10,20)
The greatest common divisor for 10 and 20 is 10.
Thus, L.C.M. (10,20) = 200gcd(10)
L.C.M. (10,20) = 20

LCM Methods
By Finding the Multiples:
The method to find the least common multiple of any given numbers is first to
write down the multiples of individual numbers and then find the first common
multiple between them. Suppose, there are two number 11 and 33. Then the multiples
of 11 and 33 can be written as;
Multiples of 11 = 11, 22, 33, 44, 55, ….
Multiples of 33 = 33, 66, 99, ….
We can see, the first common multiple or the least common multiple for both the
numbers is 33. Hence the LCM (11, 33) = 33.
By Prime Factorisation:
Another method to find the LCM of the given numbers is by prime factorization.
Suppose, there are three numbers 12, 16 and 24. Let us write the prime factors of
all three numbers individually.
12 = 2 x 2 x 3
16 = 2 x 2 x 2 x 2
24 = 2 x 2 x 2 x 3
Now writing the prime factors of all the three numbers together, we get;
12 x 16 x 24 = 2 x 2 x 3 x 2 x 2 x 2 x 2 x 2 x 2 x 2 x 3
Now pairing the common prime factors we get the LCM. Hence, there are four pairs
of 2 and one pair of 3. So the LCM of 12, 16 and 24 will be;
LCM (12, 16, 24) = 2 x 2 x 2 x 2 x 3 = 48

LCM Tree
The Least common multiple trees can be formed by using the prime factorisation
method. Suppose there are two numbers 60 and 282. Then, first let us write the
prime factors of these two numbers, such as;
60 = 6 x 10 = 2 x 3 x 2 x 5
282 = 2 x 141 = 2 x 3 x 47
Now let us represent the above prime factorization using a tree.
From the above tree diagram, we can take the pair of common factors and unique
factors from the branches of both the numbers and multiply them as a whole to get
the LCM. Therefore,
LCM (60, 282) = 2 x 2 x 3 x 5 x 47 = 2820

HCF and LCM Problems


Example 1: Prove that: LCM (9 & 12) × HCF (9 & 12) = Product of 9 and 12
Solution: LCM and HCF of 9 and 12:
9 = 3 × 3 = 3²
12 = 2 × 2 × 3 = 2² × 3
LCM of 9 and 12 = 2² × 3² = 4 × 9 = 36
HCF of 9 and 12 = 3
LCM (9 & 12) × HCF (9 & 12) = 36 × 3 = 108
Product of 9 and 12 = 9 × 12 = 108
Hence, LCM (9 & 12) × HCF (9 & 12) = 108 = 9 × 12
Example 2: 8 and 9 are two co-prime numbers. Using this numbers verify, LCM of Co-
prime Numbers = Product Of The Numbers
Solution: LCM and HCF of 8 and 9:
8 = 2 × 2 × 2 = 2³
9 = 3 × 3 = 3²
LCM of 8 and 9 = 2³ × 3² = 8 × 9 = 72
HCF of 8 and 9 = 1
Product of 8 and 9 = 8 × 9 = 72
Hence, LCM of co-prime numbers = Product of the numbers
Example 3: Find the HCF of 1225, 910, 1835, 2140
Solution: The required HCF is = HCFof12,9,18,21LCMof25,10,35,40 = 31400

2) Divisibility
Divisibility rules or Divisibility test have been mentioned to make the division
procedure easier and quicker. If students will learn the rules or tests for 1 to
20, they can solve the problems in a better way. Some of the numbers like 2, 3, 4,
5 have rules which can be understood easily. But rules for 7, 11, 13, are little
complex and need to be understood elaborately. Mathematics is not very easy for
some of us. At times, the need for tricks and shorthand techniques is felt, so as
to solve math problems faster and easier without actual calculation. It will also
help students to score better marks in exams. These rules are a great example of
such shorthand techniques.

Divisibility Rules
The rules for division by different numbers are explained here. Let us go through
one by one.
Divisibility by 1
Every number is divisible by 1. Divisibility rule for 1 doesn’t have any
particular condition. Any number divided by 1 will give the number itself,
irrespective of how large the number is. For example, 3 is divisible by 1 and 3000
is also divisible by 1 completely.

Divisibility by 2
Any even number or number whose last digit is an even number i.e. 2,4,6,8
including 0 is always completely divisible by 2.
Example: 508 is an even number and divisible by 2 but 509 is not an even number,
hence not divisible by 2. Procedure to check whether 508 is divisible by 2 or not
is as follow:

• Consider the number 508


• Just take the last digit 8 and divide it by 2
• If the last digit 8 is divisible by 2 then the number 508 is also divisible
by 2.

Divisibility rules for 3


Divisibility rule for 3 states that a number is completely divisible by 3 if the
sum of its digits is divisible by 3 i.e., it is a multiple of 3
Consider a number, 308.To check whether 308 is divisible by 3 or not, take sum of
the digits (i.e. 3+0+8= 11). Now check whether the sum is divisible by 3 or not.
If the sum is a multiple of 3 then the original number is also divisible by 3.
Here, since 11 is not divisible by 3, 308 is also not divisible by 3.
Similarly, 516 is divisible by 3 completely as the sum of its digits i.e.
5+1+6=12, is a multiple of 3.

Divisibility by 4
If the last two digits of a number are divisible by 4, then that number is a
multiple of 4 and is divisible by 4 completely.
Example: Take the number 2308. Consider the last two digits i.e. 08. As 08 is
divisible by 4, the original number 2308 is also divisible by 4.

Divisibility by 5
Numbers with last digit 0 or 5 are always divisible by 5.
Example: 10, 10000, 10000005, 595, 396524850 etc.

Divisibility by 6
Numbers which are divisible by both 2 and 3 are divisible by 6. That is, if last
digit of the given number is even and the sum of its digits is a multiple of 3,
then the given number is also a multiple of 6.
Example: 630, the number is divisible by 2 as the last digit is 0.
The sum of digits is 6+3+0 = 9, which is also divisible by 3.
Hence 630 is divisible by 6.

Divisibility rules for 7


The rule for divisibility by 7 is given below:
Example: Is 1073 divisible by 7?
• From the rule stated remove 3 from the number and double it, which becomes 6.
• Remaining number becomes 107, so 107-6 = 101.
• Repeating the process one more times, we have 1 x 2 = 2.
• Remaining number 10 – 2 = 8.
• As 8 is not divisible by 7, hence the number 1073 is not divisible by 7.

Divisibility by 8
If the last three digits of a number are divisible by 8, then the number is
completely divisible by 8.
Example: Take number 24344. Consider the last two digits i.e. 344. As 344 is
divisible by 8, the original number 24344 is also divisible by 8.

Divisibility by 9
The rule for divisibility by 9 is similar to divisibility rule for 3. That is, if
the sum of digits of the number is divisible by 9, then the number itself is
divisible by 9.
Example: Consider 78532, as the sum of its digits (7+8+5+3+2) is 25, which is not
divisible by 9, hence 78532 is not divisible by 9

Divisibility by 10
Divisibility rule for 10 states that any number whose last digit is 0, is
divisible by 10.
Example: 10, 20,30,1000,5000,60000 etc.

Divisibility rules for 11


If the difference of the sum of alternative digits of a number is divisible by 11
then that number is divisible by 11 completely.
In order to check whether a number like 2143 is divisible by 11 following is the
procedure.

• Group the alternative digits i.e. digits which are in odd places together and
digits in even places together. Here 24 and 13 are two groups.
• Take the sum of the digits of each group i.e. 2+4=6 and 1+3= 4
• Now find the difference of the sums; 6-4=2
• If the difference is divisible by 11, then the original number is also
divisible by 11. Here 2 is the difference which is not divisible by 11.
• Therefore, 2143 is not divisible by 11.

Divisibility rules for 13


For any given number, to check if it is divisible by 13, we have to add four times
of the last digit of the number to the remaining number and repeat the process
until you get a two-digit number. Now check if that two-digit number is divisible
by 13 or not. If it is divisible then the given number is divisible by 13.
For example: 2795 → 279 + (5 x 4) → 279 + (20) → 299 → 29 + (9 x 4) → 29 + 36 →65.
Number 65 is divisible by 13, 13 x 5 = 65.

3) Factor
Numbers we can multiply together to get another number.

Example: 2 and 3 are factors of 6, because 2 × 3 = 6

number can have MANY factors!

Example: What are the factors of 12?


• 3 × 4 = 12, so 3 and 4 are factors of 12
• 2 × 6 = 12, so 2 and 6 are also factors of 12
• and 1 × 12 = 12, so 1 and 12 are factors of 12 as well

So 1, 2, 3, 4, 6 and 12 are all factors of 12


And -1, -2, -3, -4, -6 and -12 also, because multiplying negatives makes a positive.

In Algebra factors can be expressions like "x+3" etc


Example: (x+3) and (x+1) are factors of x2 + 4x + 3:

Factors of a number are defined as numbers or algebraic expressions that divide a


given number/expression evenly. We can also say, factors are the numbers which
are multiplied to get another number. For example, 1, 3 and 9 are the factors of
9, because 1 × 9 = 9 and 3 × 3 = 9. Here, the concepts of factors are explained
which will help to understand how to find the factors and know the prime
factors of some common digits. Here we will discuss finding factors, formulas to
find the number of factors, product and sum of factors.

Definition of Factors of a Number


The factors of a number are defined as the number which can be multiplied to get
the original number. By multiplying two factors of a number, a product is obtained
which is equal to the original number. It should be noted that factors of any
number can be either positive or negative. For example, in the case of 6, the
factors can be 2 and 3 as 2 × 3 gives 6. Here, 2 and 3 are factors while 6 is the
product. The other factors of 6 are 1 and 6, etc. We can also consider -1, -2, -3
and -6 as the factors of 6, because when we multiply any two negative numbers, it
results in positive number, such as;
-1 × -6 = 6
-2 × -3 = 6
Hence, the factors of 6 in total are 1,2,3,6,-1,-2,-3 and -6.
But normally, the factors are considered to be only positive numbers.
Point to remember: Fractions could not be considered as factors for any number.

Factors Formulas
There are basically three types of formulas considered for factors. They are:

• Number of Factors
• Product of Factors
• Sum of Factors
Let us assume N is a natural number, for which we need to find the factors. If we
convert N into the product of prime numbers by prime factorisation method, we can
represent it as;
N = Xa × Yb × Zc
where X, Y and Z are the prime numbers and a, b and c are their respective powers.
Now, the formula for the total number of factors for a given number is given by;

• Total Number of Factors for N = (a+1) (b+1) (c+1)


The formula for the sum of all factors is given by;

• Sum of factors of N = [(Xa+1-1)/X-1] × [(Yb+1-1)/Y-1] × [(Zc+1-1)/Z-1]


The formula for the product of all factors is given by;

• Product of factors of N = NTotal No. of Factors/2


Example: Find the total number of factors of 90 along with sum and product of all
factors.
Solution: Write the prime factorisation of 90 first.
90 = 2 × 45 = 2 × 3 × 15 = 2 × 3 × 3 × 5
90 = 21 × 32 × 51
Here, X = 2, Y = 3, Z =5 and a = 1, b = 2, c = 1
Therefore, total number of factors of 90 = (a +1)(b+1)(c+1) = (1+1)(2+1)(1+1) = 2
× 3 × 2 = 12
Sum of factors of 90 = [(21+1-1)/2-1] × [(32+1-1)/3-1] × [(51+1-1)/5-1] = 3/1 ×
26/2 × 24/4 = 3 × 13 × 6 = 234
Product of factors of 90 = 90Total factors of 90/2
= 9012/2 = 906

How to Find Factors of a Number?


Knowing how to calculate factors of a number is extremely crucial in maths. The
steps to find the factors of a number are given below in a very easy to understand
way. An example is taken to make the explanation easier.

• Step 1: Choose a number (say, 16)


• Step 2: Write the common factors of 16 which will include (16 × 1), (-16 × -
1), (8 × 2), (-8 × -2), (4 × 4), and (-4 × -4).
• Step 3: Further factor the factors until a prime number is reached. In this
case, 8 can be factored further.
• Step 4: Write down all the factors again. The (8 × 2) will now become (4 × 2
× 2).
• Step 5: Write all the unique number that is obtained.
So, the factors of 16 will be 1, 2, 4, 8, 16, – 1, – 2, – 4, – 8, and – 16. Here,
the positive factors of 16 are only 1, 2, 4, 8, and 16.
Another Example:
Consider the number as 80.
80 = 10 × 4
= (5 × 2) × 8
=(5 × 2) × (4 × 2)
= (5 × 2) × (2 × 2 × 2)
Now, the factors of 80 will all the combination from 5 × 2 × 2 × 2 × 2 and 1
itself (as 1 × 80 = 80). So, the positive factors of 80 are 1, 2, 4, 5, 8, 10, 16,
20, 40, and 80. It should be noted that there will also be negative factors whose
count have to be even.

How to Calculate Factors of Large Numbers?


To calculate the factors of large numbers, divide the numbers with the least prime
number i.e. 2. If the number is not divisible by 2, move to the next prime
numbers i.e. 3 and so on until 1 is reached. Below is an example to find the
factors of a large number.
Example: 1420

Steps Prime Factors Product

Step 1: Divide by 2 2 710

Step 2: Again Divide by 2 2 355

Step 3: Divide by 5 71

In step 3, a prime number is obtained as a product and so, the process is stopped.
The factors will be all the multiples of 1, 2, 2, 5, 71, 355, 710. Now, the
positive factors of 1420 will be 1, 2, 4, 5, 10, 20, 71, 142, 284, 355, 710, and
1420.
In the same case, if only prime factors are considered, it is called the prime
factorization of that number. In this way, it is easy to factor a number and know
its factors and prime factors.

Factors of 36
From the above steps, the factors of 36 are obtained. It should be noted that 1
and 36 itself is also factors. Also, the negative multiples of each are also
considered as factors.

All Factors of 36

Positive Factors of 1, 2, 3, 4, 6, 9, 12, 18, and 36


36

Negative Factors of – 1, – 2, – 3, – 4, – 6, – 9, – 12, – 18,


36 and – 36

Factor Pairs of 36
The factor pairs of 36 include the following:

1. 1 × 36 = 36
2. 2 × 18 = 36
3. 3 × 12 = 36
4. 4 × 9 = 36
5. 6 × 6 = 36

How to Factor 36?


To calculate the factors of 36, follow the given steps:
• Step 1: Write 36
• Step 2: 36 is the square of 6 and so write 36 as 6 × 6
• Step 3: Factor 6 as 2 and 3
• Step 4: Write 36 = 2 × 3 × 2 × 3
• Step 5: Now multiply all the multiples to get all the factors of 36

Prime Factors of 36
The prime factors of 36 are the prime numbers from the list of factors for the
number 36. The prime factors of 36 can also be obtained by another method which is
explained below. Here are all the prime factors of 36:

• Prime Factors of 36 = 22 × 32

How To Calculate Prime Factor of 36?


To calculate the prime factors of 36, divide it with the least prime number which
is 2. When it cannot be divided further with two, divide it with the next prime
number which is 3 in this case till the end product is 1. This is called prime
factorization of a number and a step by step format for 36 is given below.

• Step 1: Divide 36 with 2


2 ÷ 36 = 18

• Step 2: Again divide 18 with 2


2 ÷ 18 = 9

• Step 3: Since 9 is no more divisible by 2, move to the next prime number i.e.
3
3 ÷ 9 = 3

• Step 4: Finally, divide 3 with 3 to get 1.


3 ÷ 3 = 1
From the above steps, it can be said that the prime factor of 36 will be 2 × 2 × 3
× 3 i.e. 22 × 32. It should be noted that the above-mentioned method will not only
work for 36 but even for all the large number also.
he factors of 12 and the prime factors of 12 can be seen here along with a proper explanation. This article will help you to learn how to
factor 12 quickly and also to understand the concept of factors of a number easily with the given examples.

List of Factors of 12
12 is a composite number.
Note: 1 and 12 itself is also factors. The negative multiples of each are considered as factors too.

Positive Factors of 12 Negative Factors of 12

1 -1

2 -2

3 -3

4 -4

6 -6
12 -12

Factor Pairs of 12
The factor pairs of 12 include the following:

1. 1 × 12 = 12
2. 2 × 6 = 12
3. 3 × 4 = 12
4. 4 × 3 = 12
5. 6 × 2 = 12

How to Factor 12?


To calculate the factors of 12, follow the given steps:

• Step 1: Write 12
• Step 2: 12 is a multiple of 6 and so write 12 as 6 × 2
• Step 3: Factor 6 as 2 and 3
• Step 4: Write 12 = 2 × 3 × 2
• Step 5: Now multiply all the multiples to get all the factors of 12

Prime Factors of 12
The prime factors of 12 are the prime numbers from the list of factors for the
number 12. The prime factors of 12 can also be obtained by another method which is
explained below. Here are all the prime factors of 12:

• Prime Factors of 12 = 2 X 2 X 3

Find the Factors of 12?


To calculate the prime factors of 12, divide it with the least prime number which
is 2. When it cannot be divided further with two, divide it with the next prime
number which is 3 in this case till the end product is 1. This is called prime
factorization of a number and a step by step format for 12 is given below.

• Step 1: Divide 12 with 2


• 12 ÷ 2 = 6
• Step 2: Again divide 6 with 2
• 6 ÷ 2 = 3
• Step 3: Since 3 is no more divisible by 2, move to the next prime number i.e.
3

3 ÷ 3 = 1, We get 1
From the above steps, it can be said that the prime factor of 12 will be 2 × 2 × 3
i.e. 22 X 3. It should be noted that the above-mentioned method will not only work
for 12 but even for all the large numbers like Prime Factors for 36 etc also.

4) Remainder
Remainders is a very crucial concept since numerous questions from Quantitative
Aptitude section require the concepts of remainder to solve them. Most of the
candidates have already studied this concept in their elementary schools and can
solve the related questions. Here is a lesson on Remainders to help the candidates
revise the topic in an efficient way.
What is Remainder?
Supposing a number “N” is divided by another number “x”; if the quotient obtained
is “Q” and the remainder obtained is “R”, then the number can be expressed as
N=Qx+R
For example, suppose 8 is divided by 3.
In this case, N=8, x=3. 3×2=6, which is 2 less than 8. hence Q=2 and R=(8-6)=2
Hence, 8=2×3+2.
Basic Remainder Theorem
The basic remainder theorem is based on the product of individual remainders.
If R is the remainder of an expression( p*q*r)/X, and pR, qR and rR are the
remainders when p,q and r are respectively divided by X, then it can be said that
((pR x qR x rR ))/X, will give the same remainder as given by (p*q*r)/X.
Let’s understand this with the help of some examples.
1) Find the remainder when (361*363) is divided by 12.
Steps
1) Take the product of individual remainders, i.e. 361/12|R =1 and 363/12|R= 3
2) Find the remainder when you divide that product by the number (361*363)/12|R=
(1*3)/12|R. answer= 3
This is Basic Remainder theorem put across in Numbers.
2) Find the remainder when 106 is divided by 7 i.e. (106/7)R.
Solution:
106=103x103
Thus(106/7)R = (103/7 x 103/7)R = ((6 * 6)/7)R = (36/7)R = 1.
So the remainder is 1.
“Remainder when the product of some numbers is divided by the requisite number is
the product of individual remainders of the numbers”– This is Basic Remainder
Theorem put across in words.
Concept of Negative Remainder
The remainder obtained by division of a number N by a divisor X can be expressed
in two ways as “R” and “X-R”
For example, 10/11 remainder is +10 itself. It can also be written as 10-11= -1
Similarly, 32/10 remainder is +2 or -8
Let’s express the solution for questions 31 above, in another way- based on the
concept of negative remainder. Thus(106/7)R = (103/7 x 103/7)R = ((-1 * -1)/7)R =
(1/7)R = 1.
Let’s see why this happens:
If the numbers N1,N2,N3 give remainders of R1,R2,R3 with quotients Q1,Q2,Q3 when
divided by a common divisor D
N1=DQ1+R1 N2=DQ2+R2 N3=DQ3+R3
Multiplying=N1xN2xN3
=(DQ1+R1)x (DQ2+R2)x(DQ3+R3) = D(some number)+(R1xR2xR3) =first part is divisible
by D, hence you need to check for the individual remainders only.

5) Cyclicist of Number
The first thing you need to understand is that when we multiply two integers
together, the last digit of the result depends only on the last digits of the two
integers.

For example:

24 * 12 = 288

Note here: …4 * …2 = …8

So when we are looking at the units digit of the result of an integer raised to a
certain exponent, all we need to worry about is the units digit of the integer.

Let’s look at the pattern when the units digit of a number is 2.

Units digit 2:
2^1 = 2

2^2 = 4

2^3 = 8

2^4 = 16

2^5 = 32

2^6 = 64

2^7 = 128

2^8 = 256

2^9 = 512

2^10 = 1024

Note the units digits. Do you see a pattern? 2, 4, 8, 6, 2, 4, 8, 6, 2, 4 … and so


on

So what will 2^11 end with? The pattern tells us that two full cycles of 2-4-8-6
will take us to 2^8, and then a new cycle starts at 2^9.

2-4-8-6

2-4-8-6
2-4

The next digit in the pattern will be 8, which will belong to 2^11.

In fact, any integer that ends with 2 and is raised to the power 11 will end in 8
because the last digit will depend only on the last digit of the base.

So 652^(11) will end in 8,1896782^(11) will end in 8, and so on…

A similar pattern exists for all units digits. Let’s find out what the pattern is
for the rest of the 9 digits.

Units digit 3:
3^1 = 3

3^2 = 9

3^3 = 27

3^4 = 81

3^5 = 243

3^6 = 729

The pattern here is 3, 9, 7, 1, 3, 9, 7, 1, and so on…

Units digit 4:
4^1 = 4

4^2 = 16

4^3 = 64

4^4 = 256

The pattern here is 4, 6, 4, 6, 4, 6, and so on…

Integers ending in digits 0, 1, 5 or 6 have the same units digit (0, 1, 5 or 6


respectively), whatever the positive integer exponent. That is:

1545^23 = ……..5

1650^19 = ……..0

161^28 = ………1

Hope you get the point.


Units digit 7:
7^1 = 7

7^2 = 49

7^3 = 343

7^4 = ….1 (Just multiply the last digit of 343 i.e. 3 by another 7 and you get 21
and hence 1 as the units digit)

7^5 = ….7 (Now multiply 1 from above by 7 to get 7 as the units digit)

7^6 = ….9

The pattern here is 7, 9, 3, 1, 7, 9, 3, 1, and so on…

Units digit 8:
8^1 = 8

8^2 = 64

8^3 = …2

8^4 = …6

8^5 = …8

8^6 = …4

The pattern here is 8, 4, 2, 6, 8, 4, 2, 6, and so on…

Units digit 9:
9^1 = 9

9^2 = 81

9^3 = 729

9^4 = …1

The pattern here is 9, 1, 9, 1, 9, 1, and so on…

Summing it all up:

1) Digits 2, 3, 7 and 8 have a cyclicity of 4; i.e. the units digit repeats itself
every 4 digits.

Cyclicity of 2: 2, 4, 8, 6
Cyclicity of 3: 3, 9, 7, 1
Cyclicity of 7: 7, 9, 3, 1
Cyclicity of 8: 8, 4, 2, 6
2) Digits 4 and 9 have a cyclicity of 2; i.e. the units digit repeats itself every
2 digits.

Cyclicity of 4: 4, 6
Cyclicity of 9: 9, 1
3) Digits 0, 1, 5 and 6 have a cyclicity of 1.

Unit 2 Averages and problem on ages


1) Weighted Average
Weighted average is an average in which each quantity to be averaged is assigned a
weight. These weightings determine the relative importance of each quantity on the
average. Weightings are the equivalent of having that many like items with the
same value involved in the average.
Weightings are equivalent of having many similar items with the same value
involved in the average. For instance, let x be the observations and w be the
weights of the observations, the formula of weighted average is given below.
x¯¯¯=∑ni=1wixi∑ni=1wi

Or in simple terms we can write the formula as below


WeightedAverage=SumofWeightedTermsTotalNumberofTerms

To find the weighted term, multiply each term by its weighting factor, which is
the number of times each term occurs.

Solved Examples
Example: A class of 25 students took a science test. 10 students had an average
score of 80. The other students had an average score of 60. What is the average
score of the whole class?
Solution:
Step 1: To get the sum of weighted terms, multiply each average by the number of
students that had that average and then sum them up.
80 × 10 + 60 × 15 = 800 + 900 = 1700
Step 2: Total number of terms = Total number of students = 25
Step 3: Using the formula
WeightedAverage=SumofWeightedTermsTotalNumberofTerms
=170025
=68
Answer: The average score of the whole class is 68.

Weighted Average Formula


Observe that the average can be calculated directly only if the weights of all the
factors are the same.If the values a, b, c, and d are the individual averages of
each of the groups and n1, n2, n3 and n4 are the numbers of observations or number
of values, then the Weighted Average is calculated by the below Weighted Average
Formula:
The term “weight” stands for the relative importance attached to the different
values.
1) In Class I there are 12 students of average age 20 years and in Class II there
are 16 students of average age 23 years.What is the average age of both the
classes?
Solution:
=((12 ×20)+ (16 ×23))/((12+16)) = (240+368)/28 = 608/28 = 20.7 years
Note:
➜ Average always lies between the maximum and minimum value. It will be equal to
the maximum or minimum value if all the values are equal
➜ Average is the result of the net surplus + net deficit
➜ If the value of each quantity is increased or decreased by the same quantity
“q”, then the average will also increase or decrease by the quantity “q”
respectively.
➜ If the value of each quantity is multiplied or divided by a quantity “q”, then
the resultant average will also be multiplied or divided by “q” respectively.
➜ When weights of all the quantities are the same, then the average can be
calculated directly. If the weights are different, we need to use the concept of
weighted average.

Shortcut Techniques in Weighted Average


Let us now see a useful shortcut technique to find the weighted average method
with the help of an example.
2) The average of a batsman in 16 innings is 36. In the next innings, he is
scoring 70 runs. What will be his new average?
a) 44 b) 38 c) 40 d) 48
Solution:
Conventionally solving
New average = (old sum+ new score)/(total number of innings) = ((16
×36)+70)/((16+1)) = 38
Shortcut technique:
Step 1: Take the difference between the new score and the old average = 70-36= 34
Step 2: This 34 extra run is spread over 17 innings = 34/17=2
Step 3: Hence, the average increases by 2 = >36+2 = 38 is the new average
Let us try another question using the same technique.
3) The average marks of 19 children in a particular school is 50. When a new
student with marks 75 joins the class, what will be the new average of the class?
Solution:
1) Take the difference between the old average and the new marks = 75-50=25
2) This score of 25 is distributed over 20 students => 25/20 = 1.25
3) Hence, the average increases by 1.25=> 50+1.25 = 51.25 Let us try another
question where the average dips.
4) The average age of Mr. Mark’s 3 children is 8 years. A new baby is born.Find
the average age of all his children?
Solution:
The new age will be 0 years. The difference between the old average and the new
age = 0-8= -8
This age of 8 years is spread over 4 children => (-8/4= -2) Hence, the average
reduces to 8-2= 6 years
Now let us look at a technique which will help us compute the new value when the
average is given.
Take this question for example.
5) The average age of 29 students is 18. If the age of the teacher is also
included the average age of the class becomes 18.2.Find the age of the teacher?
a) 28 b) 32 c) 22 d) 24
Solution:
Conventionally solving
Let the average age of the teacher =x
(29 × 18 + x × 1)/30
Solving for x, we get x = 24
Using the shortcut, based on the same method we used previously
Step 1: Calculate the change in average = 18.2-18 = 0.2
This change in 0.2 is reflected over a sample size of 30
New age is an increased by 30 * 0.2 = 6 years above the average 18+6= 24.
Think Quest- The average age of 26 students in an MBA school is 30. One student
among these quits the school in between. Can you find the age of that student if
the new average is 29.8?

Illustrations on Weighted Average


6) Average goals scored by 15 selected players in EPL is 16.Maximum goals scored
by a player is 20 and minimum is 12.Goals scored by players is between 12 and 20.
What can be the maximum number of players who scored at least 18 goals?
a) 10 b) 5 c) 9 d) 6 e) none of these
Solution: Option (c)
To maximize the number of players who scored 18 and above number of goals, we will
assume that only one person has scored 20. To counter him, we will have one person
who will score 12 goals.
15-2 =13 players left
Now to maximize the 18 and above goals,for every two players who are scoring 18,
we will have one player scoring 12.This is done, to arrive at the average of 16.We
will have 8 players with a score of 18 and 4 players with a score of 12.The last
player will have a score of 16 Thus, the maximum number of people with 18 and more
goals = 9.
7) The average weight of a group of 8 girls is 50 kg. If 2 girls R and S replace P
and Q, the new average weight becomes 48 kg.The weight of P= Weight of Q and the
weight of R= Weight of S.Another girl T, is included in the group and the new
average weight becomes 48 kg. Weight of T= Weight of R. Find the weight of P?
a) 48 kgs b) 52 kgs c) 46 kgs d) 56 kgs
Solution: Option (d)
8 x 50 +R+S-P-Q= 48×8 R+S-P-Q=-16
P+Q-R-S= 16 R=S and P=Q
P-R=8 One more person is
included and the weight = 48 kg
Let the weight be a = (48 ×8+a)9/9 = 48
a=48 kg= weight of R weight of P= 48+8= 56 kg

2) Allegation
Mixture and alligation is an integral topic in the Quantitative section of
the CAT exam. There are a lot of questions asked every year from this topic and
thus, is very crucial from the examination point of view. So, here is an
explanation of the Mixture and Alligation along with various illustrations to help
the candidates learn the topic with ease.
Definitions of Mixture and Alligation
Here are some important definitions related to mixtures and alligation that would
help to tackle the related questions with ease.
Mixtures
When two or more components are mixed in a certain ratio, a mixture is created.
Types of mixtures:
Simple Mixtures:- When two or more different ingredients are mixed together, a
simple mixture is formed.
Compound Mixtures:- When two or more simple mixtures are mixed together, a
compound mixture is formed.
Alligation:
To understand the concept of alligations, let’s take an example and look at it
from all angles possible.
Example 1:
30 boys with an average weight of 60 kgs and 20 girls with an average weight of
40.Find the average weight of the whole class? Find the average weight of the
whole class?
Solution:
To find the answer to the above question, one can use the formula for weighted
average. Suppose the question was framed a bit differently, the Average weight of
boys is 60, the average weight of girls is 40 and average weight of the whole
class is 52.
Find the ratio between the number of boys and girls
Solution:
The fastest way to solve this question would be by using Alligation. Read below
for its solution using alligation.
So what exactly is Alligation?
It is the reverse of weighted average; i.e. If the averages of two groups are
separately given and the average of the whole group is given, then we can find out
the ratio between the groups.
For example, in the above question, the data given is-
Average weight of boys=60, average weight of girls=40 and average weight of the
whole class is =52
The data can be represented in an Alligation chart as given below and use it to
find the ratio between the number of boys and girls in the class.

Take the difference across: x= 60-52=8 and y= 52-40=12. That is how we get the
ratio between the boys and girls as 12:8 or 3:2.Let us look at another question
where you can use
Let’s look at another question where you can use Alligation.
Question:
In what ratio should a shopkeeper mix two types of rice, one costing 20 rupees/kg
and another costing 10 rupees/kg to get a rice variety costing 14 rupees/kg

Here also we can use Alligation as follows:


x=14-10=4 y=20-14=6 The ratio between the type 1 and type 2 rice is 4:6 or 2:3
Formulae In Mixtures & Alligations
There is no necessity to memorize all the formula given here, as all of them can
be easily deduced with the help of logic.
Formula A,B and C are just algebraic representations of Alligation.

• (Quantity of cheaper entity )/(Quantity of expensive entity) = (Price of


expensive entity – Mean price )/(Mean Price- Price of cheaper entity)

• Cost of Mixture(Cm)
Quantity of Component A =xa Quantity of Component B = xb Cost of Component A= Ca
Cost of Component B= Cb
Then, Cm= (x_a C_a+x_(b ) C_(b ))/(x_a+ x_b )

• If you have 2 mixtures (M1 and M2), each having two components A and B
M_1 contains A & B in the ratio p:q M_2 contains A & B in the ratio ? r:s

Quantity of A and B in the final mixture (xa & xb)

• If a vessel contains “a” liters of liquid A; and ”b” liters are withdrawn and
replaced by another liquid B of equal quantity and the operation is repeated
n times, then:
1) (Liquid A left after nth operation)/(Initial quantity of A in the vessel) = (a-
b)n/(a)n
2) (Liquid A left after nth operation )/(Liquid B left after nth operation) = ((a-
b)n/(a)n )/(1- (a-b)n/(a)n )
Applications Of Mixtures
Consider the following common data to understand the application better.
A= 20 B=10 Na =30 Nb= 45 The answer will be 14, in each of the following cases.
1) Average of a class
Example:
Class A has 30 students scoring average marks of 20 and Class B has 45 students
scoring average marks of 10.What is the average of both classes together (14).
2) Average price of goods
Example:
Sunita buys 30 kgs of sugar at Rs 20/kg and 45 kgs of sugar at 10/kg.
What is the average price? (Rs 14/kg).
3) Percentage
Example:
An antique article salesman makes a profit of 20% by selling 30% of his articles
and a profit of 10% by selling 45% of his goods.
What is his net percentage profit? (14%).
4) Average Speed
Example:
A bus travels at 20kmph for 30 km and at 10 kmph for 45 km.
Find the average speed of the bus for the entire journey (14kmph).
5) Mixtures
A dishonest milkman dilutes two milk cans such that milk can A containing 30
liters is mixed with 20% water and milk can B containing 45 liters is mixed with
10% water. He combines both mixtures. Find the percentage of water in the final
mixture (14%).
Application of Alligation
A) Application in profit & loss questions
Question: How many must kgs of rice costing Rs 8 per kg be mixed with 36 kg of
rice costing Rs 5.40 per kg so that 20% gain may be obtained by selling the
mixture at Rs 7.20 per kg?
a) 10 kg
b) 12 kg
c) 10.8 kg
d) 8 kg
Solution:
Always remember that we need to take only the cost price and not the selling price
when we consider the price of all entities in the question.In the question, the
selling price of the mixture is given. We need to find out the cost price using
the profit percentage given=20%. Cost price × 1.2=7.2 , implies that cost price of
the mixture is 6.
From the Alligation chart,
X=6-5.40=0.6 Y=8-6=2
Thus the ratio between the quantity of type A rice and type B rice is 0.6:2 or
3:10 Therefore => x=10.8 kg.
B) Application in mixtures of liquids
Question: Vessel A contains milk and water in the ratio 4:5. Vessel B contains
milk and water in the proportion 5:1.In what proportion should quantities be taken
from A & B to form a mixture in which milk and water are in the ratio 5:4?
a) 2:5
b) 3:2
c) 2:3
d) 5:2
Solution:
For this question, let’s consider the proportion of milk in each mixture. In
Vessel A, the proportion of milk in 4/((4+5) )=4/9.in vessel B, the proportion of
milk is 5/(5+1) = 5/6.

The amount of milk in the mixture= 5/((5+4)) = 5/9


X=(5/6)-(5/9)=5/18
Y=(5/9)-(4/9)=1/9
The ratio is (5/18):(1/9)=5:2
C) Application in percentage questions
Question: A person has Rs.5000. He invests a part of it at 3% per annum and the
remainder at 8% per annum simple interest. His total income in 3 years is Rs.750.
Find the sum invested at different rates of interest?
a) 3500, 1500
b) 1000, 4000
c) 3000, 2000
d) 1800, 3200
Solution:
He is investing part of it at 3% at annum and remaining at 8% per annum Since he
is getting Rs 750 as profit for 3 years, rate percent of the whole amount is 5%
That can be calculated from
PNR/100 = 750 ((5000 ×3 ×R))/100 = 750 R= 5%
From the Alligation chart,
The ratio of investment will be 3:2. Look at the answer options for that ratio.
Only option (c) (3000, 2000) gives that ratio.

3)Mean Median and mode

Mean, Median and Mode


Mean – It is the sum of values of all the observations divided by the total number
of observations.
The mean for grouped data is calculated as follows:
Direct Method: x¯=∑fixi∑fi
Assumed Mean Method: x¯=a+∑fidi∑fi
Step Deviation Method: x¯=a+(∑fiui∑fi)h
Median – It is a measure of central tendency which gives the value of the middle-
most observation in the data.Medianof a grouped data is calculated using the
formula
Median=l+(n2−cff)×h

Mode – It is the value among the observations which occurs most frequently. The
mode for a grouped data is found using the formula
Mode=l+(f1−f02f−1−f0−f2)h

Class 10 Maths Chapter 14 Statistics Chapter Questions

1. The following table shows the ages of the patients admitted to a hospital
during a year:

Age in Years 15-25 25-35 35-45 45-55

Number of Patients 6 11 21 23

Find the mean of the data given above


Solution:
The class mark is found using the formula
ClassMark=Upperclasslimit−Lowerclasslimit2

Age Number of Patients (fi) Class Mark (xi) fixi


15-25 6 20 120

25-35 11 30 330

35-45 21 40 840

45-55 23 50 1150

∑fi=61 ∑fixi=2440

The mean is calculated as follows:


x¯=∑fixi∑fi

Substituting the values, we get


x¯=244061=40

The mean is 40.


Stay tuned to BYJU’S to get the latest notification on SSC exam along with AP SSC
model papers, exam pattern, marking scheme and more.

Mean Median Mode Formula


Me The Mean, Median and Mode are the arithmetic average of a data set. This is
found by adding the numbers in a data set and dividing by how many numbers there
are. The median is the middle number in a data set when the numbers are listed in
either ascending or descending order. The mode is the value that occurs the most
often in a data set, and the range is the difference between the highest and
lowest values in a data set.
The Mean
x¯¯¯=∑xN

Here,
∑ represents the summation
X represents scores
N represents number of scores.
The Median
If the total number of numbers(n) is an odd number, then the formula is given
below:
Median=(n+12)thterm

If the total number of the numbers(n) is an even number, then the formula is given
below:
Median=(n2)thterm+(n2+1)thterm2

The Mode
Themodeisthemostfrequentlyoccuringscoreorvalue.

Solved Examples
Question: Find the mean, median, mode, and range for the following list of
values: 13, 18, 13, 14, 13, 16, 14, 21, 13
Solution:
Given sequence: 13, 18, 13, 14, 13, 16, 14, 21, 13
The mean is the usual average, so:
13+18+13+14+13+16+14+21+139=15
Note that the mean isn’t a value from the original list. This is a common result.
You should not assume that your mean will be one of your original numbers.
The median is the middle value, so to rewrite the list in order:
13, 13, 13, 13, 14, 14, 16, 18, 21
There are nine numbers in the list, so the middle one will be
9+12=102=5
= 5th number, so, the median is 14.
The mode is the number that is repeated more often than any other, so 13 is the
mode.
The largest value in the list is 21, and the smallest is 13, so the range is 21 –
13 = 8.
mean: 15
median: 14
mode: 13
an Median Mode Formula

Unit 3 Percentages, Profit and loss


1)Basics of Percentages
Percentage formula is used to find the amount or share of something in terms of 100. In its simplest form, percent means per
hundred. To express a number between zero and one, percentage formula is used. It
is defined as a number represented as a fraction of 100. Denoted by the symbol =
%, the percentage is majorly used to compare and find out ratios.

Formula to Calculate Percentage


The Percentage Formula is given as,

Percentage = (Value ⁄ Total Value) × 100

Solved Example Questions Using the Formula for Percentage:


Question 1: Express ⅘ as percentage.
Solution:
⅘ = (4 × 20 ⁄ 5 × 20)
= (80 ⁄ 100)
= 80%
Question 2: There are 150 students in a class. Out of them, 75 are girls. Find the
percentage of girls in the class?
Solution:
Total students in the class=150
Girls in the class = 75
% of girls in the class = (75 ⁄ 150) × 100 = (7500 ⁄ 150) = 50%
e percentage concept is required in several quantitative aptitude topics to solve the related questions. Also, questions directly related to
percentages holds good weightage and thus, the this topic is considered as one of the most important topics for the CAT. Learn the
percentages for CAT easily along with detailed explanations and solved examples here.

Definition of Percent:
“Per cent” means for every hundred. If you get 25 marks in an exam where the full marks is 50, your percentage marks will be what you
will get in a total of hundred marks. (25/50× 100)
Using unitary method: In 50 you get 25; So in 1 you get 25/50 and in 100 you get 2550× 100 = 50 percent. Thus, what you get out of
hundred becomes the percentage.
For a fraction, if the denominator is hundred, it is called a percentage and the numerator of the fraction is called rate per cent.

Percentage for Comparison:


Percentage helps us to compare between different fractions when the denominator or the total number is different in each case. It is one
of the simplest tools for the comparison of data.
Take, for example, this table below which shows the marks obtained by a student in 3 different subjects

Subject Marks Obtained

History 60

Math 25

English 45

From this data alone, we cannot compare the marks obtained for the various subjects.. Now suppose we have the data of the total marks
obtained as follows

Subject Marks Obtained Total Marks Marks Obtained/total marks * 100

History 60 100 60%

Math 25 25 100%

English 45 90 50%

Now as all the three subjects are represented on a scale of 100, it is easy to compare the marks for the three subjects and decide which
subject has the student scored the maximum in.
Presentations of Percentages:
1) a% of b
a% of b ( a percent of b) = b% of a
a% of b is represented mathematically as (a×b)/100.
Example: 24% of 25 = [24×25/100]= 6 25% of 24 = [(25×24)/100]= 6.

2) What percentage of a is b
This is represented as (b/a× 100)
Eg. What percentage of 75 is 25? (25/75)×100 = 33.33%

Conversions Related to Percentages:


1) To convert a fraction into a percentage
Multiply and divide by 100.
Keep the denominator as 100, the numerator you obtain is the required answer.
Take for example
21/22=(21/22 x100)/100=(2100/22)/100 = 95 6/11%.

2) To find the fraction equivalent of a percentage


Divide by 100(after removing the percentage sign) e.g. ) 11 3/8%=91/8×100=91/800
eg) 25% = 25/100=1/4.

Illustration:
Question: Recently I went to buy a laptop for myself. The dealer said that he has
laptops of two companies: HP and Lenovo. He was selling the HP laptop for Rs.
42000 and told me that he will offer me the same at 7/8 of that price while the
Lenovo laptop was for Rs. 46000 and he were offering it at ?4/5?^th of that price.
I decided to buy the laptop on which I was getting a better percentage discount.
Which one should I buy?
Solution:
HP: 7/8 means (7×100)/8 = = 87.5% means a discount of 100 – 87.5 = 12.5%
LENOVO: 4/5 means 4×100/5 = 400/5= 80 % means a discount of 100 – 80 = 20%. So I
should buy the LENOVO laptop.

The Concept Of Change:


There are two types of change
1. Absolute value change
It is the actual change in the quantity. For example, if there are 10 rabbits in
the first year and 15 rabbits in the second year, the absolute change in the
number of rabbits is 5
2. Percentage change
This can be obtained by calculating the absolute change and dividing it by the
initial number of rabbits present e.g. ) percentage change = (Absolute value
change)/(Original quantity)×100= 5/10=1/2×100= 50%.

Percentage Change and Percentage Point Change


If the pass percentage of a class was 75 % in 1991 and 85% in 1992, we can
calculate the percentage point change and the percentage change as follows
Percentage point change= Final percentage – Initial percentage= 85 %– 75% = 10
percentage points Percentage change= (Final percentage – Initial
percentage)/(Initial percentage)×100 = (85 – 75)/75 x 100 = 40/3%.
• To Increase a Number By a Given Percent
Use the formula: (100 + rate)/100
Example: Increase 40 by 20%
Here the rate = 20 Multiply the number by the above formula to get the answer 40 x
((100 + 20))/100=120×40/100 = 48. (We are effectively computing(40+20/100×40)
It is easier to remember that given the base value=x, the final value after
increase can be found as:
20% increase= (x+0.2x= 1.2x); 30% increase= (x+0.3x=1.3x); 5% increase=
(x+0.05x=1.05x).
• To Decrease a Number By a Given Number
Use the formula (100 – Rate)/100
Example: Decrease 40 by 20%
Here the rate = 20. Multiply the number by the above formula to get the answer
40 x ((100- 20))/100=80×40/100=32. (We are effectively computing (40-20/100×40))
It is easier to remember that given the base value=x, the final value after
decrease can be found as:
20% decrease= 0.8 x (0.8= 1-0.2); 30% decrease= 0.7x (0.7= 1-0.3); 5% decrease=
0.95x (0.95= 1-0.05).

To find the percentage increase or decrease of a given number, the concept of


percentage change is used as explained above.
1) Percentage Increase = (Final Value – Initial Value)/(Initial Value) = (Total
Increase )/(Initial Value) x 100%
2) Percentage Decrease = (Initial Value – Final Value)/(Initial Value) = (Total
Decrease)/(Initial Value) x 100%

Illustration:
Question: In the IPL match an analysis was done for the two openers of both teams:
Delhi Daredevils (DD) and Deccan Chargers (DC). It was found that the two DC
openers Gilchrist and Gibbs together scored 20% more than what the two DD openers
Sehwag and Gambhir scored. Gambhir scored 30 and Gibbs scored 40. How much
percentage less/more did Sehwag score than Gilchrist if Gilchrist scored 80?
Solution:
Say runs scored by Sehwag = a, Gambhir = b, Gilchrist = c and Gibbs = d.
Then (c + d) = 1.2 (a + b)
Also b = 30, d = 40, c = 80
80 + 40 = 1.2 (a + 30) => 120/1.2 = a + 30 => a = 70
So Sehwag = 70. So Sehwag scored (80 – 70)/80 x 100 = 10/80 x 100 = 12.5 % less
than Gilchrist.
Note: Here we took 80 as the denominator because we need to calculate how much
Sehwag scored less “than” Gilchrist. So, Gilchrist’s score has to be made the
denominator as it is the base value that the difference is being compared to.

Basic Formulae related to Percentage Change:


a) If the price of a commodity increases by r%, then the reduction in consumption
so as to not increase the expenditure is r/(100+r) x 100.This formula can also be
used to compare incomes between two people.
e.g.) If P’s income is r% more than Q’s income, then Q’s income is less than P’s
income by r/(100+r) x 100.

b) If the price of a commodity decreases by r%, then the increase in consumption


so as to not decrease the expenditure is r/(100-r) x 100 . This formula can also
be used to compare incomes between two people.
Example: If P’s income is r% less than Q’s income, then Q’s income is greater than
P’s income by r/(100-r) x 100.

c) If there is a consecutive percentage change of a% and b%, the net change can be
calculate as
Net change= a + b + ab/100

Illustrations:
3. Anil is taking an examination which has two sections A and B. He starts with
section A and when he proceeds to section B he realizes that the marks carried for
each question has decreased by 40% as compared to section A. How many more
questions (in percentage) should he solve in section B as compared to section A if
he wants to score same marks in section A and section B?
Solution:-
Given that both sections carry equal marks. Solution: Here marks per question
decreases by 40%, so using the formula (b), he should solve ((40/(100-40))×100)% =
66.66% more questions in section B if he wants to score same marks.

4. The petrol prices shot up by 20% due to Iraq War. Amit travels 2000 kms every
month and his car gives a mileage of 20 kms per liter. By how many kilometers
should he reduce his travel to maintain his expenditure to the previous level.
Solution:
At 20 kms per liter he bought 2000/20= 100 liters of petrol. Now the price the
increased by 20%; using formula (a), the consumption has to be reduced by
(20/(100+20))×100) % = 16.66%.
So, he should reduce his travel by (16.66×2000)/100= 333.33 kms to maintain the
same expenditure level.
5. After receiving two successive pay hikes if Ashish’s salary becomes 15/8 of his
initial salary. By how much percent was the salary raised the first time if the
second raise was twice as high (in %) as the first?
a) 15 %
b) 20%
c) 25%
d) 30%
Solution:
This question can be solved using two approaches:
Approach 1: (conventional)
Suppose first raise was of a % then the second raise = 2a % Now, using formula (c)
net change: a + b + ab/100?a + 2a + 2axa/100
Now if initial salary was X, then % change = ((15 X)/8– X)/X×100=7/8× 100
So 3a + (2a^2)/100 = 700/8 solving we get a = 25%. So, the first raise was of 25%.
Approach 2: (Recommended)
In these types of questions it is quicker to go from the answer options. First
calculate the net change= ( (15 X)/8– X )×100 = 700/8% = 87.5 %
Start with option (b) 20% First year raise 20% next year raise= 40% Net change
(use formula (c)) 20 + 20 + 400/100 = 44% With option (d) Net change = 30 + 60 +
1800/100 = 108% This way, you will get option (c) 25% is the correct option.
4)Basic of profit and loss

Profit in Maths is considered as the gain amount from any business activity.
Whenever a shopkeeper sells a product, his motive is to gain some benefit from the
buyer in the name of profit. Basically, when he sells the product more than its
cost price, then he gets the profit on it but if he has to sell it for less than
its cost price, then he has to suffer the loss.
The concept of profit and loss is basically defined in terms of business. Any
financial benefit gained in business goes to the owner of the business.

Profit Formula
Profit is explained better in terms of cost price and selling price. Cost price is
the actual price of the product or commodity and selling price is the amount at
which the product is sold. So, if the selling price of the commodity is more than
the cost price, then the business has gained its profit. Therefore formula to
calculate the profit is;
Profit or Gain = Selling Price – Cost Price
But, when the product is sold at selling price lesser than the cost price, it is
termed as loss. Therefore,
Loss = Cost Price – Selling Price

Profit Percentage
Once the profit is calculated we can also derive the percentage profit e have
gained in any business by the formula given here;
P% = (P/CP) × 100
Where P is the profit and CP is the cost price.

Types of Profit
There are three types of profit used in business. They are:

1. Gross Profit
2. Operating Profit
3. Net Profit

Gross Profit
Gross profit is the amount gained by any business or company after removing the
cost associated with the making and selling of the product from the selling price.
The revenue yielded in the company’s income after sales of the commodity should be
reduced by the amount or cost it took to make the product or provide any service
to the customer’s, to get the gross percentage of the profit.

Operating Profit
A business’s operating profit tells what is the contribution of the company’s
operations to its profitability. The operating profit is basically the ratio of
operating income and sales revenue.
Operating Profit = Operating Income/Sales Revenue

Net Profit
Net profit includes all the cost amount generated by the business as revenue. It
represents the actual sum of money made by any business.
Companies examine all three types of profit with the help of profit margin. In
such case, the profit, whether gross, operating, or net, is divided by the return.
It exhibits how well the business uses its earnings. A large ratio means it makes
a lot of profit for each revenue. A low ratio means the business’s costs are
consuming into its profits. Ratios vary according to each trade.

How to Calculate Profit?


To calculate the profit gained by any business, follow the steps below:

• Determine the cost price of the products sold.


• Now calculate the total selling price of the products sold.
• Subtract the cost price and selling price, to get the profit amount.
• To calculate the profit margin, divide the profit amount with cost price.
• Multiply the profit margin with 100 to get in percentage.

Profit Examples
Problem 1: If a shopkeeper sells Apple at Rs.200 per kg, whose cost price is
Rs.150/- per kg. Then find the profit gained by the shopkeeper.
Solution: Given Cost Price = Rs.150/-
And Selling Price = Rs.200/-
From the formula of profit, we know,
Profit = Selling Price – Cost Price
P = 200 – 150
P = 50
Therefore, the shopkeeper gains Rs.50/- from the business.
Problem 2: Find the gain percentage for the above example.
Solution: By the profit percentage formula, we know,
P% = (P/CP) × 100
Since, P = 50 and CP = 150
Therefore,
P% = (50/150) × 100
P% = 100/3

Problem 3: Sanju sold a digital camera for Rs.5,000, on which he gains 25%. What
is the cost price of the camera?
Solution: For the digital camera: Gain = 25%.
Let cost price (C.P.) = Rs.100.
Therefore, selling price (S.P.) = (100 + 25) = 125
When selling price (S.P.) is Rs.125, cost price (C.P.) is Rs.100.
Therefore, when selling price (S.P.) is Rs.5000,
cost price (C.P.) = 100/125 × 5000 = (100 × 5000)/125 = 500000/125 = 4000
Therefore, cost price (C.P.) of the digital camera = Rs. 4000.

profit and loss is a topic from the CAT quantitative ability section and almost 3-
4 questions are asked every year in the exam. To help the candidates prepare the
profit and loss topic, here are all the related details, formulae and solved
examples.

Terminologies Involved in Profit & Loss:


1) Cost Price (CP):
The amount paid to buy a product or the cost involved in manufacturing a product
is called the Cost Price (CP) of that product.

2) Selling Price (SP):


The price at which an article is sold to the customer.

3) Marked Price (MP):


The price which the shopkeeper fixes for the product (the price on the label) in
anticipation of some discount they may be asked by the customer for the product.

4) Discount:
The reduction made on the marked price is called the discount. When no discount is
given, the Selling Price = Marked Price.

5) Margin:
When profit percentage is calculated as a percentage of SPM then it is known as
margin.
6) Types of cost:

a) Fixed Cost➜ The kind of cost which is fixed in all cases


b) Variable cost➜ The cost which varies according to the number of units produced
c) Semi-Variable cost➜ Cost which is fixed for some entities and variable for
some entities.

Important:
Profit and Loss is always calculated on Cost Price.
%Profit = Profit/CP x100 = (SP-CP)/CP x 100 = ((SP/CP)– 1)x100
% Loss= Loss/CP x100 = (CP-SP)/CP x100 = ((1-SP/CP) )x100
Discount is always calculated on Marked Price.
% Discount= Discount/MP x100.

Other Profit & Loss Formulae Involved:


(a) Profit = SP-CP (SP>CP)
(b) Loss= CP-SP (CP>SP)
(c) SP= MP- discount
(d) MP = SPx100/(100 – discount%)
(e) If two items are sold at the same price, one with a profit of x% and another
with a loss of x%, there will be an overall loss given by x2/100.
(f) If the CP of two items is the same, and one is sold at x% gain, other at x %
loss, then the net loss or net profit=0.
(g) On an offer of buy “a” goods and get “b” goods free, i.e., if “a+b” articles
are sold at the rate of “a” articles,
then the percentage discount = [(b/(a+b))×100]
(h) If there are successive discounts of x% and y%, the effective discount can be
calculated as [x+y – (xy/100) ]%
(i) There are many shopkeepers who cheat by using false weights, then the gain to
the shopkeeper on using the false weight can be calculated as follows (100 +
Gain%)/100=(True Weight)/(False Weight).
Explanation:
Suppose the shopkeeper is using a false weight in such a way that when his meter
shows X kg (False weight) he is actually selling Y kg (True weight). Also say that
the price he is selling the article is Rs. Z per kg. Thus, when his meter is
showing X kg, he is actually selling Y kg. So for the shopkeeper, the cost price
(CP) = YZ
But the selling price (SP) = XZ
So, his profit = (SP – CP)/CP=(XZ – YZ)/YZ = X/Y– 1or (X/Y– 1)x100 %
➜(Profit%)/100=X/Y– 1
Illustrations on Profit & Loss Shortcut Tricks:
1) Monte Carlo is offering a summer discount of 20% on all the sweaters. Ram buys
sweaters which cost him Rs. 3000/- and then resold them in his shop at a discount
of 10% on the market price. Find the total profit he earned.
Solution:
When Ram bought sweaters from Monte Carlo he paid Rs. 3000.Thus, 3000 is Ram’s
Cost Price Using MP = SPx100/(100 – discount%)
MP = (3000 x 100)/(100 – 20) = 3000 x 100/80 = 3750. Rs 3750 is now, the new
Marked Price
He sold all sweaters at a discount of 10%; discount = 10 x 3750/100 = Rs. 375.
So he sold the sweaters at Rs. 3750 – Rs. 375 = Rs. 3375. Profit = Rs. (3375 –
3000) = Rs. 375.

2) In the previous question, calculate the profit percentage earned by Ram.


Solution:
Profit earned = Rs. 375 Cost Price (CP) for Ram = Rs. 3000 So profit percentage =
375 ×100/3000 = 12.5%.

3) The CP of 40 articles = SP of 50 articles. What is the profit or loss


percentage?
Solution:
Since the Cost Price of 40 articles= Selling Price of 50 articles, we realize that
the transaction is a loss for the shopkeeper.
CP will be greater than SP
CP/SP=50/40=5/4
Loss percentage = (1-SP/CP)x100
Loss Percentage = (1-4/5)=1/5 x 100= 20%.

4) A shopkeeper sells two shirts at the same price; one at a discount of 20% and
another at a gain of 20%.Find the overall loss or the gain he earned.
Solution:
As both shirts are sold at the same price one at 20% loss and other at 20%
profit, he will make a net loss of 202/100= 4%

Profit and Loss Definitions


Cost Price: The price (amount) paid to purchase a product or the cost incurred in
manufacturing a product is known as the cost price (CP) of that product.
Types of cost
1. Fixed cost: It is a type of cost which is fixed under all conditions and does
not vary according to the number of units produced.
2. Variable cost: Variable cost is a type of cost which varies according to the
number of units. This is quite easy to understand.
3. Semi-variable cost: As the name suggests, these costs are the ones that are
fixed in part and variable in part. Effectively, this is the case that we see most
often.Imagine the scenario in a factory. There is a capital cost, which remains
the same under all conditions (fixed cost) and a variable cost of the product,
which in turn depends upon various factors.
Selling Price: The price at which a product is sold is called the selling price
(SP) of the product.
Marked Price: Do shopkeepers put up price on the label that they wish to sell on
or they put up an inflated price? If you think closely, majority of the
shopkeepers mark-up their products, in anticipation of the discounts they would
have to offer. This is a clever way of operating. Mark-up the price in advance,
offer a discount and make the customer feel happy, and then sell the product.
Pretty effective, eh?
List Price: List price or the tag price is the price that is printed on the tag of
the article. For all practical purposes, we assume it to be same as the marked-
price.
Margin: The profit percentage on selling price is known as MARGIN.
Example 1: Pranav sell an article at a discount of 80% and get a profit of 60% on
that article calculate the mark up over the cost price?
Solution:
Let us put the above definitions in use.
Let us assume that Cost Price = Rs 100.
So, Selling Price = Rs 160.
Now, after giving a discount of 80% over MP, Rs 160 is the SP.
Let the Marked Price: MP
SP = 20% of MP
160 = 20% of MP
MP = 800
% Mark Up = (700/100) × 100 = 700 %.
Things to remember in this question:
Mark Up over Cost Price = Marked Price – Cost Price = M
% Mark Up Cost Price = [{M/CP} x 100]

Mark Up over Selling Price = Marked Price – Selling Price = M


% Mark Up Selling Price = [{M/SP} x 100]
Focus closely on what is being asked. Remember Profit and Loss is nothing else but
an application of Percentages and basic calculation skills.
Tooltip 2: Profit and Loss Basic Concepts
1. One can generate a profit only if Selling Price> Cost Price
2. One generates a loss when Selling Price < Cost Price.
3. Profit = Selling Price – Cost Price
% Profit = [{(Selling Price -Cost Price)/Cost Price} x 100]
4. Loss = Cost Price – Selling Price
% Loss = [{(Cost Price-Selling Price)/Cost Price} x 100]
Tooltip-3: Profit and Loss Multiplying equivalents to figure out the sale price
and cost price
Equivalent for Sale Price:
If there is a profit of P %,
Cost Price = CP
Then, SP = {(100+P)/100} x CP
If there is a loss of L %,
Cost Price = CP
Then SP = {(100-L)/100} x CP
Equivalent for Cost Price:
If there is a profit of P %,
Cost Price = CP
Sale price= SP
Then,
CP = [{100/(100+P)} x SP]
If there is a loss of L %,
Then
CP =[{100/(100+L)} x SP]
Example-1: Mehak and Pranav sells some article for Rs 8000 each. Mehak calculates
her profit per cent on his CP and Pranav calculates his profit per cent wrongly on
SP. What is the difference in their actual profit if both claim to have a profit
of 60%?
Solution:
For Mehak
SP = Rs 8,000
Profit = 60% of CP
CP = Rs 5000
Profit = Rs 3000
For Pranav
SP = Rs 8,000
Profit = 60% of SP = 60% of 8000 = 4800
CP = Rs 3,200
Profit = Rs 4800
So, the difference in profit = Rs 1800
Example-2: Megha and Richa sold two articles at Rs 12,000 each. One is sold at a
profit of 20% and another one at a loss of 20%. What is the net loss?
Solution:
For article-1
SP 1 = Rs 12,000
CP1 = Rs 12,000/1.2 = Rs 10,000
For article-2
SP2 = Rs 12,000
CP2 = Rs 12,000/0.8 = Rs 15,000
So, total CP = Rs 25,000 and total SP = Rs 24,000
So, loss = Rs 1,000.

5) Profit and loss Percentage


IMPORTANT FACTS
Cost Price:
The price, at which an article is purchased, is called its cost price, abbreviated
as C.P.
Selling Price:
The price, at which an article is sold, is called its selling prices, abbreviated
as S.P.
Profit or Gain:
If S.P. is greater than C.P., the seller is said to have a profit or gain.
Loss:
If S.P. is less than C.P., the seller is said to have incurred a loss.
IMPORTANT FORMULAE
1. Gain = (S.P.) - (C.P.)
2. Loss = (C.P.) - (S.P.)
3. Loss or gain is always reckoned on C.P.
4. Gain Percentage: (Gain %)
Gain x 100
Gain % =
C.P.

5. Loss Percentage: (Loss %)


Loss x 100
Loss % =
C.P.

6. Selling Price: (S.P.)


SP = (100 + Gain %) x C.P
100
7. Selling Price: (S.P.)
(100 - Loss %)
SP = x C.P.
100

8. Cost Price: (C.P.)


100
C.P. = x S.P.
(100 + Gain %)

9. Cost Price: (C.P.)


100
C.P. = x S.P.
(100 - Loss %)

10. If an article is sold at a gain of say 35%, then S.P. = 135% of C.P.
11. If an article is sold at a loss of say, 35% then S.P. = 65% of C.P.
12. When a person sells two similar items, one at a gain of say x%, and the other
at a loss of x%, then the seller always incurs a loss given by:
Common Loss and Gain % 2 x 2
Loss % = = .
10 10

13. If a trader professes to sell his goods at cost price, but uses false
weights, then
Error
Gain % = x 100
(True Value) - (Error) %.

Problem 1 :

Cindy bought 50 pens for $100. She then sold each pen for $2.50. Find the profit or loss percentage.

Solution :

Cost price of 50 pens = $100

Cost price of 1 pen = 100/50 = $2

Selling price of 1 pen = $2.50

Because the selling price of 1 pen is more than cost price of 1 pen, there is profit.

Finding Profit :

Profit = Selling price - Cost price

Profit = 2.50 - 2

Profit = 0.50

Finding Profit Percentage :

Profit % = (Profit/Cost price) ⋅ 100 %


Profit % = (0.50/2) ⋅ 100 %

Profit % = 25 %

Problem 2 :

Jacob purchased a house for $49,000. He spent $6000 for repair and $5,000 for air-conditioning. If he had
sold the house $58,000, find the gain or loss percentage in this transaction. (If it is needed, round your
answer to the nearest hundredths)

Solution :

Total amount spent on the house is

= 49,000 + 6,000 + 5,000

= 60,000

This is the cost price of the house ($60,000).

Selling price of the house = $58,000

Because the selling price of the house is less than the cost price, there is loss.

Finding Loss :

Loss = Cost price - Selling price

Loss = 60,000 - 58,000

Loss = 2,000

Finding Loss Percentage :

Loss % = (Loss/Cost price) ⋅ 100 %

Loss % = (2000/60000) ⋅ 100 %

Loss % = 3.33 %

Problem 3 :

Goods are purchased for $1500. If one fifth of the goods sold at a profit of 5% and the remaining four-fifth
of the goods at a profit of 10%, find the net profit percentage.

Solution :

Cost price of one-fifth of the goods is


= 1/5 ⋅ 1500

= 300

Selling price of one-fifth of the goods (at 5% profit) is

= (100 + 5)% of 300

= 105% of 300

= 1.05 ⋅ 300

= 315

Cost price of remaining four-fifth of the goods is

= 4/5 ⋅ 1500

= 1,200

Selling price of the remaining four-fifth of the goods (at 10% profit) is

= (100 + 10)% of 1200

= 110% of 1200

= 1.10 ⋅ 1200

= 1,320

Selling price of the total goods

= 315 + 1,320

= 1,635

Finding Net Profit :

Net profit = S.P of total goods - C.P of total goods

Net Profit = 1635 - 1500

Net Profit = 135

Finding Net Profit Percentage :

Net profit % = (Net profit/Cost price) ⋅ 100 %

Net profit % = (135/1500) ⋅ 100 %


Net profit % = 9 %

Problem 4 :

A trader bought a product for $200. If marks his goods 20% above the cost price and gives a discount of 10%
for cash, find his profit percentage

Solution :

Cost price of the product = $100

Marked price (20% above the cost price) is

= (100 + 20) % of 200

= 120% of 200

= 1.2 ⋅ 200

= 240

Selling price price is the price which is after 20% discount from the marked price.

So, the selling price is

= (100 - 10) % of Marked price

= 90% of 240

= 0.9 ⋅ 240

= 216

Finding Profit :

Profit = Selling price - Cost price

Profit = 216 - 200

Profit = 16

Finding Profit Percentage :

Profit % = (Profit/Cost price) ⋅ 100 %

Profit % = (16/200) ⋅ 100 %

Profit % = 8 %
Problem 5 :

A person wants to get 20% profit after selling his object at 20% discount. Find the required percentage
increase in marked price.

Solution :

Let the cost price be $100.

Then, the selling price (at 20% profit) is

= (100 + 20)% of 100

= 120% of 100

= 1.2 ⋅ 100

= 120

Selling price price is the price which is after 20% discount from the marked price.

It has been illustrated in the picture given below.

From the above picture, we get

Selling price = (100 - 20)% of Mmarked price

120 = 80% of x

120 = 0.8 ⋅ x

Divide both sides by 0.8

120/0.8 = x

1200/8 = x

150 = x
So, the marked price is $150.

Here,

Cost price = $100

Marked Price = $150

Hence, the required percentage increase is 50%

Unit4 Ratio and Portion ,mixture and allegations Partnership

1)Basic

What Is a Ratio?

In mathematics, a ratio is a comparison of two or more numbers that indicates


their sizes in relation to each other. A ratio compares two quantities by
division, with the dividend or number being divided termed the antecedent and the
divisor or number that is dividing termed the consequent.

Example: you have polled a group of 20 people and found that 13 of them prefer
cake to ice cream and 7 of them prefer ice cream to cake. The ratio to represent
this data set would be 13:7, with 13 being the antecedent and 7 the consequent.

A ratio might be formatted as a Part to Part or Part to Whole comparison. A Part


to Part comparison looks at two individual quantities within a ratio of greater
than two numbers, such as the number of dogs to the number of cats in a poll of
pet type in an animal clinic. A Part to Whole comparison measures the number of
one quantity against the total, such as the number of dogs to the total number of
pets in the clinic. Ratios like these are much more common than you might think.

Ratios in Daily Life

Ratios occur frequently in daily life and help to simplify many of our
interactions by putting numbers into perspective. Ratios allow us to measure and
express quantities by making them easier to understand.

Examples of ratios in life:

• The car was traveling 60 miles per hour, or 60 miles in 1 hour.


• You have a 1 in 28,000,000 chance of winning the lottery. Out of every
possible scenario, only 1 out of 28,000,000 of them has you winning the
lottery.
• There were enough cookies for every student to have two, or 2 cookies per 78
students.
• The children outnumbered the adults 3:1, or there were three times as many
children as there were adults.

How to Write a Ratio

There are several different ways to express a ratio. One of the most common is to
write a ratio using a colon as a this-to-that comparison such as the children-to-
adults example above. Because ratios are simple division problems, they can also
be written as a fraction. Some people prefer to express ratios using only words,
as in the cookies example.

In the context of mathematics, the colon and fraction format are preferred. When
comparing more than two quantities, opt for the colon format. For example, if you
are preparing a mixture that calls for 1 part oil, 1 part vinegar, and 10 parts
water, you could express the ratio of oil to vinegar to water as 1:1:10. Consider
the context of the comparison when deciding how best to write your ratio.

Simplifying Ratios

No matter how a ratio is written, it is important that it be simplified down to


the smallest whole numbers possible, just as with any fraction. This can be done
by finding the greatest common factor between the numbers and dividing them
accordingly. With a ratio comparing 12 to 16, for example, you see that both 12
and 16 can be divided by 4. This simplifies your ratio into 3 to 4, or the
quotients you get when you divide 12 and 16 by 4. Your ratio can now be written
as:

• 3:4
• 3/4
• 3 to 4
• 0.75 (a decimal is sometimes permissible, though less commonly used)

Practice Calculating Ratios With Two Quantities

Practice identifying real-life opportunities for expressing ratios by finding


quantities you want to compare. You can then try calculating these ratios and
simplifying them into their smallest whole numbers. Below are a few examples of
authentic ratios to practice calculating.

1. There are 6 apples in a bowl containing 8 pieces of fruit.


1. What is the ratio of apples to the total amount of fruit? (answer: 6:8,
simplified to 3:4)
2. If the two pieces of fruit that are not apples are oranges, what is the
ratio of apples to oranges? (answer: 6:2, simplified to 3:1)
2. Dr. Pasture, a rural veterinarian, treats only 2 types of animals—cows and
horses. Last week, she treated 12 cows and 16 horses.
1. What is the ratio of cows to horses that she treated? (answer: 12:16,
simplified to 3:4. For every 3 cows treated, 4 horses were treated)
2. What is the ratio of cows to the total number of animals that she
treated? (answer: 12 + 16 = 28, the total number of animals treated.
The ratio for cows to total is 12:28, simplified to 3:7. For every 7
animals treated, 3 of them were cows)

Practice Calculating Ratios With Greater Than Two Quantities

Use the following demographic information about a marching band to complete the
following exercises using ratios comparing two or more quantities.

Gender

• 120 boys
• 180 girls

Instrument type

• 160 woodwinds
• 84 percussion
• 56 brass

Class

• 127 freshmen
• 63 sophomores
• 55 juniors
• 55 seniors

1. What is the ratio of boys to girls? (answer: 2:3)

2. What is the ratio of freshmen to the total number of band members? (answer:
127:300)

3. What is the ratio of percussion to woodwinds to brass? (answer: 84:160:56,


simplified to 21:40:14)
4. What is the ratio of freshmen to seniors to sophomores? (answer: 127:55:63.
Note: 127 is a prime number and cannot be reduced in this ratio)

5. If 25 students left the woodwind section to join the percussion section, what
would be the ratio for the number of woodwind players to percussion?
(answer: 160 woodwinds – 25 woodwinds = 135 woodwinds;
84 percussionists + 25 percussionists = 109 percussionists. The ratio of the
number of players in woodwinds to percussion is 109:135)

Ratio and Proportions is another important topic from banking exam point of view; it can come as an
individual question and it can also come as a part of data interpretation.

RPs (short for Ratio and Proportions) are easy enough if we could just get the hang of the basic concept.
Today my effort will be to revise the basic premise of RPs and tackle some short cut concepts too.

Basic Concepts

1. Ratio
When two numbers are represented in the form of another; this is done by expressing one number as a
fraction of another.

Thus, we have a:b; where a is the antecedent, and b is the consequent (a little general knowledge
doesn't hurt even in math!)

Thus when we write 4:16,


it can be re-written as 2:8 and further simplifying it can be said to be 1:4.

Which actually means, the number ‘4’ is 4 times to get the figure 16.

And all of you know, that 1:4 can also be written as ¼.

2. Ratios to percentages:
This 1/4 ratio can be denoted as a percentage too! It’s 25% How?

Well, 1/4 x 100 = 25% [1/4th is also known as one quarter, that is one part out of 4 parts.]

2/4 = 1/2 =50%

3/4 = 75%

and, 4/4 = 100%

3. Ratios to Degrees:
Supposing, we have A:B:C:D, being four farmers, who have contributed Rs. 25,000,
Rs. 75,000, Rs. 65,000 and Rs. 35,000 respectively.

Using all their contributions, they have purchased a land, which surprisingly is
circular! (C’mon Math need not be boring!)

They decided that they would all receive a part of the circular land based on
their contribution; how will they divide the circular land? So one of them who had
completed his class 12, suggested they divide the land on a pie chart model!

Total contribution = Rs. 2,00,000 (25000+75000+65000+35000)


Ratio of contribution = 25000:75000:65000:35000
= 25:75:65:35 (always cancel off the 000s
first!)
= 5:15:13:7 (this is our last stage, where no
more common factors

are possible, where total


of the ratios is 40)

We know, if we need A’s share, then A’s ratio = 5/40,


B’s = 15/40,
C’s = 13/40 and D’s = 7/40.
Total area = 360°, then A’s share of the total area/ share of 360° = 360 x 5/40 =
45°
B’s share = 360 x 15/40 = 135°
C’s share = 360 x 13/40 = 117° and D’s = 360x7/40 = 63°

4. Comparing two ratios or fractions:


If you are give two fractions, say 5/3 and 7/8, and we need to find out which
fraction is greater than the other what do we do?

We can either calculate their percentages, 5/3x100 = 167% and 7/8x100 = 88%
(approx values)
So we know, that 5/3 is the greater fraction!
Another method can be to compare the two fractions. And to compare we have to make
their denominators equal.

To make their denominators equal, for the first fraction, we multiply 8 to both
the numerator and denominator. Therefore the first fraction = 40/24.
The second fraction, by multiplying 3 to make the second fraction = 21/24.

Obviously 40 is greater than 21!

Hence, 40/24 or 5/3 is greater than 21/24 or 7/8!

5. Proportions
Proportions is where two ratios are compared and equated.

Where a:b is a ratio and c:d is another ratio, and if they are equal,
then, they can be re-written as a:b :: c:d. { the ‘::’ sign means ‘equal to’}

Therefore, a:b = c:d

1:2 :: 7:14 , try out the simplifications!

‘a’ and ‘d’ are called extremes as they are in the extreme ends! And ‘b’ and ‘c’
are called means as they are in the middle!

6. Properties of proportions

(i) a:b = c:d


can be written as a/b = c/d

which implies, axd = bxc, or, ad = bc


this property helps in solving many questions

(ii) if a:b = b:c, which means this proportions between three numbers is in the
form of

‘continued proportions’, as all three numbers are having a connection.

so, a/b=b/c or,


ac=bxb or,
ac=b2

MIXTURE:

When two or more components are mixed together then it is known as Mixture

Mixture is of two types they are:


1.Simple Mixture:When two or more different ingredients are mixed together,a
simple mixture is formed

2.Compound Mixture:When two or more simple mixtures are mixed together, a compound
mixture is formed.

ALLIGATION:

It enables us to find the ratio in which two or more ingredients at the given
price must be mixed to produce a mixture of desired price.

Where M=Mean Value which is between X and Y, X=Cheaper Price, Y=Price higher than
the Mean Value

Before Looking Into Mixture And Alligation Look Into Ratio And Proportions.

TYPE 1:

BASED ON THREE INGREDIENTS OR THREE RATIO’S:

1.The Grocer sells apples of 3 varieties whose price per kg’s are Rs.40,Rs.60 and
Rs. 90 .If he sells 1kg of apple by missing all these 3 varieties for Rs.80 what
is the ratio by which these varieties are to be mixed?

Explanation:
1st variety 3rd variety 2nd variety

1 4

2 1

Ratio’s by which these 3 Qualities are missed are=1:2:4


2.Tea worth Rs.126 per kg and Rs.135 per kg are mixed with a third variety in
the ratio 1:1:2.If the mixture is worth Rs.153 per kg, the price of the third
variety per kg will be?

Explanation:

Given that the tea worth Rs.126 and Rs.135 per kg is mixed in the ratio 1:1.

Therefore price for 1kg= = Rs.130.50

Now we have two price 130.50 and the price of the third variety is X
The price of the third variety=Rs.175.50
3.There are three vessels of equal capacity holds milk and water in the ratio of
1:2,2:3 and 1:4if the content of all the three vessel s are mixed in asingle
vessel then.Find the ratio of milk and water in the new vessel?

Explanation:

Here there are three ratio’s

1:2=3Part

2:3=5Part

1:4=5Part

Lcm =15Litres

Milk Water

5Litres 10Litre

6Litre 9Litre

3Litre 12Litre

14Litres 31Litre

The new Ratio is 14:31

TYPE 2:

BASED ON REMOVING AND ADDING SOME QUANTITY :

1)How much water must be added to 60 litres of milk at 1.5 litre for Rs.20,So as
to have a mixture worth Rs.10 2/3 per litre?

Explanation:

C.P for 1.5 litre of milk =Rs.20

C.P for 1 litre of milk=Rs.40/3

C.P for 1 litre of water=Rs.0

Given Mean value=Rs.32/3


8/3 : 32/3= 1:4

Therefore water in the mixture in 60 litres is (1/5)*60=12 litres


2)In a 729 litres mixture of milk and water ,the ratio of milk to water is 7:2.To
get a new mixture containing milk and water in the ratio 7:3,the amount of water
to be added is?

Explanation:

729

7x
2x

9x=729 x=81

Old Mixture New Mixture

7:2 7:3

Already the mixture contains water in the proportion 2x and the new mixture
contains water in the proportion 3x .So x quantity of water is added to the
mixture .Therefore 81 litres of water is added to the new mixture.
3)8 litres are drawn from a cask filled with wine and is then filled with water
.This operation is performed three more times.The ratio of the quantity of wine
now left in the cask to that of the total solution is 16:81.How much wine does the
cask originally ?

Explanation:

When the final amount of solute that is not replaced calculated as:

Initial Amount * (Volume After Removal/Volume After Replacing)^n

Final ratio of solute not replaced to total is

Initial Ratio * (Volume After Removal/Volume After Replacing)^n

Here let us assume the initial value is 1 then Wine .Let the Quantity of wine in
the cask originally be x

Therefore

1* [(x-8)^4/x]
=[16/81]

x=24litres

Wine originally in the cask is 24 Litres


4)A vessel contains mixture of liquids A and B in the ratio 3:2.When 20 litres of
the mixture is taken out and replaced by 20litres of liquid B,the ratio changes
to 1:4.How many litres of liquid A was there initially present in the vessel?

Explanation:

Original ratio=3:2

3/5 * (5X-20)/5x=1/4

X=6 then the liquid A in the vessel =3X=3*6=18Litres

Unit:-5 simple Insert and compound Insert

1) Basic concept

Interest rates are very powerful and intriguing mathematical concepts. Our banking
and finance sector revolves around these interest rates. One minor change in these
rates could have tremendous and astonishing impacts over the economy. But why?

Before determining the reason of this why? Let’s first know what is interest and
these interest rates?

Interest is the amount charged by the lender from the borrower on the principal
loan sum. It is basically the cost of renting money. And, the rate at which
interest is charged on the principal sum is known as the interest rate. The rate
at which interest is charged depends on two factors

1. The value of money doesn’t remain same over time. It changes with time. The
net worth of ₹ 100 today will not be same tomorrow i.e. If 5 pens could be
bought presently with a INR 100 note then in future, maybe only 4 pens can be
bought with the same ₹ 100 note. The reason behind this the inflation or
price rise. So, the interest rate includes this factor of inflation
2. The credibility of the borrower, if there is more risk and chance of default
on borrower’s part then more interest will be charged. And, if there is less
chance of payment failure on the part of borrower then the rate of interest
would be lower.

The above two reason becomes the basis of why interest rates are so important and
have a great effect on markets and economy. Since a minor rise in interest rates
increases the cost of borrowing for the borrower and as a result, he has to pay
more interest on his loan amount and thus, a decline in his money income that he
could spend on other products which create a ripple effect of decreased spending
throughout the economy and vice versa. Since change in interest rate has a chain
effect in the market, it has a great deal of importance in the study of market,
finance, and economy. And that’s why, forms an integral part of the curriculum in
the MBA programs. But, a relatively simpler level of questions is asked in
the CAT based on the concepts learned at the time of high school.

These concepts are categorized into type of interests

• Simple Interest
• Compound Interest

Let’s first start and understand Simple Interest because as the name suggests it
is simple and comparatively easy to comprehend.

Simple interest is that type of interest which once credited does not earn
interest on itself. It remains fixed over time.

The formula to calculate Simple Interest is

SI = {(P x R x T)/ 100}

Where, P = Principal Sum (the original loan/ deposited amount)

R = rate of interest (at which the loan is charged)

T = time period (the duration for which money is borrowed/ deposited)

So, if P amount is borrowed at the rate of interest R for T years then the amount
to be repaid to the lender will be

A = P + SI

Consider a basic example of SI to understand the application of above formula such


as Find the simple interest on ₹ 68000 at 16 2/3 % p.a. for 9 months.

Here, P = ₹68000

R = 162/3 % = 50/3% p.a.

T = 9 months = 9/12 years = ¾ years

SI = (68000 x 50/3 x ¾ x 1/100) = ₹8500


Some useful results based on Simple Interest

RESULTS APPLICATION SOLUTION

In this case, r1 = 6%,


T1 = 2 years

Adam borrowed some r2 = 9%, T2 = 3 years


money at the rate of 6%
p.a. for the first two r3 = 14%, T3 = 4 years
If rate to interest is r1% for
(since, beyond 5 years
T1 years, r2% for T2 years …. years, at the rate of
rate is 14%)
rn for Tn years for an 9% p.a. for the next
investment. And if the Simple three years, and at the and Simple interest =
Interest obtained is ₹a on the rate of 14% p.a. for ₹11400
investment. Then the principal the period beyond 5
amount is given by years. If he pays a Therefore, P = (11400 x
total interest of ₹ 100)/ (6*2 +9*3 +14*4)
a x 100/ (r1T1 + r2T2 + …+ rnTn) 11400 at the end of
nine years, how much = 1140000/ (12 + 27 +
money did he borrow? 56)

= 1140000/ 95

= ₹12000

Let x be his capital or


principal.

Therefore, R = (1/3 x *
0.07 +¼ x *0.08
If a person deposits sum of ₹A A man invested 1/3 of +5/12 x*0.10)/x
at r1% p.a. and sum of ₹B at his capital at 7%; ¼ at
8% and the remainder at R = (1/3 * 0.07 +¼ *0.08
r2% p.a. then the rate of +5/12 *0.10)
interest for whole sum is 10%. If his annual
income is ₹561, What is R = 0.08496
R = {(Ar1 + Br2)/ (A + B)} his capital?
Total SI = ₹561

₹561 = 0.08496x

x = ₹6602

It’s a very easy


question you just need
If a sum of money becomes “n” to use this formula and
times in “T years” at Simple The rate at which a sum you will directly reach
Interest, then the rate of becomes 4 times of to an answer.
interest p.a. is itself in 15 years at
S.I. will be? Therefore, R = (100 x
R = 100(n – 1) / T 3)/15

= 20%
If a certain sum of money is
Here, T1 = 6, T2 =
lent out in n parts in such a
10 and T3 = 12
manner that equal sum of money
years resp.
is obtained at simple interest A person invests money
on each part where interest in three different And, R1 = 10%, R2 =
rates are R1, R2, … , schemes for 6 years, 10 12%, and R3 = 15% resp.
Rn respectively and time years and 12 years at
periods are T1, T2, … , 10%, 12% and 15% Simple Hence, the ratio of his
Tn respectively, then the Interest respectively. investment will be
ratio in which the sum will be At the completion of
divided in n parts can be each scheme, he gets 100/60 : 100/120 :
given by 100/180
the same interest. What
is the ratio of his 1/6 : 1/12 : 1/18
1/R1T1: 1/R2T2:..:1/RnTn investment?
1 : 1/2 : 1/3

6 : 3 : 2

Compound Interest

This the most usual type of interest that is used in the banking system and
economics. In this kind of interest along with one principal further earns
interest on it after the completion of 1-time period. Suppose an amount P is
deposited in an account or lent to the borrower that pays compound interest at the
rate of R% p.a. Then after n years the deposit or loan will accumulate to:

P(1+R/100)n

Consider this example, if an amount of 100 is deposited in saving bank account for
3 years at the interest rate of 6% p.a. Then, after one year the ₹100 will
accumulate to ₹106. Since in compound interest, interest itself earns interest,
therefore, after 1-year interest for the 2nd will be calculated on ₹106 unlike to
that of Simple interest where interest will be calculated on ₹100 only. Thus,
after the end of the third year the total amount will become ₹100(1.06)3 =
₹119.1016.

IMPORTANT FORMULAS

• When the interest is compounded Annually:

n
Amount= P (1 + R/100)

• When the interest is compounded Half-yearly:

Amount = P (1 + (R/2)/100)2n

• When the interest is compounded Quarterly:

Amount = P (1 + (R/4)/100)4n
• When the rates are different for different years, say R1%, R2% and R3% for 1
year, 2 years and 3-year resp. Then,

Amount = P (1 + R1/100) (1 + R2/100) (1 + R3/100)

• Present worth of ₹ x due n years hence is given by:

Present worth = x/ (1 + R/100)n

• If a certain sum becomes “x” times in n years, then the rate of compound
interest will be

R = 100(x1/n – 1)

• If a sum of money P amounts to A1 after T years at CI and the same sum of


money amounts to A2 after (T + 1) years at CI, then

R = (A2 – A1)/ A1 x 100

Miscellaneous Examples of application of Compound Interest

Question 1: A man invests ₹ 5000 for 3 years at 5% p.a. compounded interest


reckoned yearly. Income tax at the rate of 20% on the interest earned is deducted
at the end of each year. Find the amount at the end of third year.

Sol: Here, P = ₹5000, T = 3 years, r = 5%

Therefore, Interest at the end of 1st year = 5000 (1 + 0.05) – 5000 = ₹250

Now Income tax is 20% on the interest income so the leftover interest income after
deducing income tax = (1 – 0.2) * 250 = ₹200

Total Amount at the end of 1st year = ₹5000 + 200 = ₹5200

Interest at the end of 2nd year = 5200 (1 + 0.05) – 5200 = ₹260

Interest income after Income tax = 0.8 * ₹260 = ₹208

Total Amount at the end of 2nd year = ₹5200 + 208 = ₹5408

Interest at the end of 3rd year = ₹5408 (1.05) – 5408 = ₹270.4

Interest income after Income tax = 0.8 * ₹270.4 = ₹216.32

Total Amount at the end of 2rd year = ₹5408 + 216.32 = ₹5624.32

Question 2: A sum of ₹12000 deposited at compound interest becomes double after 5


years. After 20 years, it will become?

Sol: The rate of interest at which ₹12000 doubles after 5 years is given by

R = 100(x1/n – 1)
= 100(21/5 – 1)

=100 x (1.1486 – 1)

= 100 x 0.1486 = 14.86%

Therefore, after 20 years it becomes,

A = ₹12000(1 + 14.86/100)20

= ₹12000 (1.1486)20

= ₹12000 x 15.97

= ₹ 191671.474

Compound Interest Installments

Let a person takes a loan from bank at r% and agrees to pay loan in equal
installments for n years. Then, the value of each installment is given by

P = X/ (1 + r/100)n………X/ (1 + r/100)2 + X/ (1 + r/100)

For better understanding, let’s understand with the help of example.

One can purchase a flat from a house building society for ₹ 55000 or on the terms
that he should pay ₹ 4275 as cash down payment and the rest in three equal
installments. The society charges interest at the rate 16% p.a. compounded half-
yearly. If the flat is purchased under installment plan, find the value of each
installment.

Sol: The cost of the flat is ₹ 55000. Now, if the person could either buy flat by
paying ₹55000 or through installment plan. Since the flat was purchased through
installment plan then the loan amount = ₹55000 – 4275 (down payment) = ₹50725.

Here r = 16% compounded Half-yearly in 3 equal instalments. Let x be the amount of


installment. Then,

₹50725 = x/ (1 + 16/200)3 + x/ (1 + 16/200)2 + x/ (1 + 16/200)

₹50725 = x (1/1.2591 + 1/1.1664 + 1/1.08)

₹50725 = x (0.79421 + 0.85722 + 0.9259)

₹50725 = x (2.577)

₹50725/2.5777 = x

x = ₹19683

The above examples are just few types based on compound interest, there could be
numerous others complex and difficult questions that could come in the CAT exam.
Compound Interest is very significant topic in today’s world. It has vast and
diverse application. In exam you could also find problems that involve both simple
and compound interest. You will get thorough and fluent in this topic with time
and practice.

Definition of the term Interest: Interest is actually one of the most fundamental
business terms, and without it, the financial trading of the world would come to
standstill. Interest is defined as the “time value of money”.
What exactly does this mean?
Well, look at this way: with time, the value of money changes. Suppose you have
Rs. 100 in the year 2000. Would it still be Rs. 100 or would the amount have
grown? If you had deposited the money in a saving bank account, say with an annual
rate of interest 4%, that money would have definitely grown by now. Can you
calculate the amount you would have with you in 2013? Well, in case you can’t
right now, go through these concept notes and you would know the answer.
The concept of simple and compound interest is especially applicable to the world
of banking and economics. Whenever we borrow a certain sum of money (known as the
principal), we pay back the original amount accompanied with a certain amount of
interest on that amount. In a way, those are the charges of borrowing that sum of
money. Simple interest is one method of determining the amount due at the end of
loan duration. Another method of interest application is compound interest, but we
study about it in next article.
Simple Interest Tooltip1: The Definitions
Principal (P): The original sum of money loaned/deposited. Also known as capital.
Interest (I): The amount of money that you pay to borrow money or the amount of
money that you earn on a deposit.
Time (T): The duration for which the money is borrowed/deposited.
Rate of Interest (R): The percent of interest that you pay for money borrowed, or
earn for money deposited
Simple Interest Tooltip 2: The Formula

Where:
P: Principal (original amount)
R: Rate of Interest (in %)
T: Time period (yearly, half-yearly etc.)
Amount Due at the end of the time period, A = P (original amount) + SI

If you have a close look, Simple Interest is nothing else but an application of
the concept of percentages.
Simple Interest Tooltip 3: Basic Problems to explain the concept
Basic Problem 1: What is the SI on Rs. 7500/- at the rate of 12% per annum for 8
years?
Using the Basic Formula:
Simple Interest (SI) = (P x R x T)/100
P – Principal amount, T- Number of years, R – Rate of Interest
Given P = 7500, T = 8 Years, R = 12%
Simple Interest (S.I.) = (7500X12X8)/100
Simple Interest (S.I.) = 7200
Basic Problem 2: A man borrowed Rs 15000/- at the rate of 24% SI and to clear the
debt after 6years, much he has to return:
Using the Basic Formula:
Simple Interest (SI) = (P x R x T)/100
P – Principal amount, T- Number of years, R – Rate of Interest
Given P = 15000, T = 6 Years, R = 24%
Simple Interest (S.I.) = (15000X24X6)/100= Rs 21600
Therefore, total interest = 21600
Total repayment = S.I + Principal amount = 21600 + 15000 = Rs 36600
Basic Problem 3: A man borrowed Rs.12000 at the rate of 10% SI, and lent the same
sum toanother person at the rate of 15% what will be the gain after 5 years?
Using the Basic Formula:
Simple Interest (S.I.) = (P x R x T)/100
P – Principal amount, T- Number of years, R – Rate of Interest
The man borrowed at 10% and he lent the same sum to another person at 15%
Therefore, his gain is actually equal to the different in the interest rate (per
year)
= 15 – 10 =5% for 1 year
Thus, to calculate his gain, we use this difference as the rate of interest.
Given T = 5 years and P = Rs. 12000
Amount Gained = (12000x5x5)/100 = Rs 3000
Therefore, his gain = Rs 3000/-

EMI calculation
What is EMI Calculator?

EMI is an acronym used for Equated Monthly Installment which is an amount paid
every month by the borrower to the lender. EMI consist of two parts one is the
principal amount and the other is the interest which is calculated and paid every
month.

EMI that is paid remains fixed however, the amount paid toward principal and
interest changes as months goes by. In the EMI paid the interest is higher than
the principal amount and as months goes by and loan progresses the principal
amount gets decreased as a result of the interest also decreases. This process
continues till the entire amount is paid off.

The duration of EMI generally differs from case to case. It can be as short as
three months to two/seven years. Sometimes it can also go for more than 10 years.

The EMI calculator is a device which helps one to calculate the EMI that need to
be paid out every month. In order to calculate the EMI, there is some information
that we need to have such as:

(i) Total cost of the product

(ii) Interest rate charged for EMI and lastly

(iii) The tenure within which one has to finish making the full payment of the
product purchased.

This information helps one to understand the monthly amount that had to be paid to
the lender and also how much from the total EMI goes to principal amount and
interest. There is another point that needs to be looked at is generally the
interest rates charged by lender come with two options; one is floating interest
rate and the second one is fixed interest rate. Sometimes the lender gives the
option for the borrower to choose what rate at which interest need to be
calculated. Floating rate is beneficial if the rates fluctuate and if there is a
decreasing trend as lesser interest only need to be paid. However, it is equally
risky if the interest rates go high leading to paying a high interest. Hence it is
safer to opt for the fixed interest rate which remains same irrespective of the
market condition.

How to use EMI Calculator?

The EMI calculator is simple and easy to operate. There are three things that are
required to calculate the EMI, they are the (i) P= principal or the loan amount,
(ii) I- Interest rate per month and finally (iii) N= the number of instalments.
With the help of this information, one can calculate by feeding the details in the
EMI calculator. Generally, the EMI comes has a lesser amount if the duration of
the loan taken is more but if the duration is smaller than the amount will be
more.

One can decide the duration to be taken based on his or her ability to repay the
money. The EMI is calculated based on the formula;

EMI= [P x I x (1+I)^N]/[(1+I)^N-1]

However while using the calculator one need not use this formula rather just enter
the right figures in the given section and one a click of the button the EMI
amounts are generated for the complete loan period.

The EMI calculator displays the following information when calculating they are;

All years for which the loan is taken

Principal amount

Interest

Total payment (principal + interest)

Balance the loan amount after each EMI and lastly

Percentage of the loan paid out of the total loan.

This chart without a doubt given every borrower an understanding on how the EMI is
calculated and how much of interest they pay out every month for the loan money
they have taken. This will surely make the borrower create an awareness has to how
to manage his or her monthly income and plan out the EMI payments. Sometimes the
EMI calculator also shows a graphical depiction of the same information both in
pie charts and in bars for easy reference. Every other bank comes with EMI
calculator which anyone of us can use it on our own as it is that simple to fill
in the figures and calculate the amount.

Floating Rate EMI Calculation

When taking Equated Monthly Installment there is always an interest associated


with based on which it is calculated. This interest comes with two options, i.e.
(i) Fixed rate and (ii) Floating rate. The borrower is generally giver the option
to choose between the type of rate based on which the EMI has to be calculated.
Floating rate is otherwise known as adjustable or flexible or variable rate. The
interest rate under this category fluctuates based on the market- lending rate.
Market lending rate or otherwise known as Marginal Cost of Lending Rate (MCLR) is
based on multiple factors including the repayment ability. The benchmark of MCLR
is generally revised regularly by the banks according to the market conditions.

In this case, the borrower has both risk and benefit in it. If the market lending
rates go down then the rate of interest calculated will be lower, as a result, the
borrower pays lesser interest but it can also happen the reverse. If the rate of
interest goes really high then the borrower is at the disadvantage of paying
higher interest and if this is the scenario then the EMI will also increase as a
result it becomes very expensive.

Fixed rate is much simpler were a standard rate is fixed at the time borrowing of
the money and that remains the same throughout the loan period irrespective of the
MCLR. Unlike floating rate, there is no uncertainty in fixed rates.

As mentioned the floating rate has it is own advantages and disadvantages.


Generally, this floating and fixed interest rate option come when a person is
availing for the home loan. One generally needs to analyse the future situation
before taking a decision to go for a floating rate otherwise, it is always wise to
go for fixed ass there is no uncertainties involved and can avoid risk.
Unit 6 Time and work

Basic concept

Time and Work Concepts


Time and work problems deal with the simultaneous performance involving the
efficiency of an individual or a group and the time taken by them to complete a
piece of work. Work is the effort applied to produce a deliverable or accomplish a
task.
A certain amount of time (T) is taken to complete a certain work (W). The number
of units of work done per unit time is called the rate of work (R). Hence, Work
(W) = Rate (R) Time (T)
Whenever some work is done, the total work itself can be taken as one unit. Hence,
we assume the total work done as one unit in the problems we encounter in order to
simplify the computations. In these cases, R = 1 / T or T = 1 / R. In other words,
R and T are inversely proportional as RT = W, which is a fixed quantity.

Time and Work Formulas


Before we plunge into time and work formulas, let’s quickly go through the
different terms that make up time and work formulas

• If A can do a piece of work in n days, then A’s one day’s work = 1/n
• If A’s one day’s work = 1/n, then A can finish the work in n days.
• If A is thrice as good a workman B, then

1. The ratio of work done by A and B = 3:1


2. The ratio of time taken by A and B to finish work = 1:3

• Total work = No of days * Efficiency.


• If a group of people are given salary for a job they do together, their
individual salaries are in the ratio of their individual efficiencies if
they work for the same number of days. Otherwise, salaries are divided in
the ratio of units of work done.
Approach for Time and Work Problems
Now that we know the terms used in time and work concepts, let’s establish the
relationship between them. This will help you to tweak time and work formulas as
per the need of the questions. Questions on Time and Work Formulas usually fall in
one of the following categories:

1. The relation between days taken by individuals to complete a given work


independently and to complete while working simultaneously or alternately.
2. Teams of men, women, children and time taken by the teams to complete work
independently or while working simultaneously.

Most questions asked are standard types of questions. Also, the approach to solve
questions is a very standard one. There are two approaches, which are exactly the
same but seem to be different – an approach using the per day’s work and
approaching LCM. The two approaches are best explained through an example.

Time and work Example


If A does a work in 10 days and B does the same work individually in 12 days, in
how many days will the work be completed if they work simultaneously?

Working with different efficiencies | Time and Work Formulas


Problems discussed in this section are based on time and work formulas we have
discussed above.

Approach 1: Per day’s work


If A can complete the work in ‘x’ days and B can complete the same work in ‘y’
days, when they work together, the time taken to complete the work is given
below.
A can complete the work in ‘x’ days. So in one day, he will do 1/x of the work. B
can complete the work in ‘y’ days. So in one day, he will do 1/y of the work.
Total work done by both in one day = (1/x) + (1/y). Hence, the total time required
to do the work = (xy)/(x +y) days.
Answer:
Since A completes the entire work in 10 days, A does 1/10th of the work in 1 day.
Since B completes the entire work in 12 days, B does 1/12th of the work in 1 day.
Working simultaneously, they do 1/10 + 1/12 = 11/60 of the work in 1 day. Thus
total days taken by both working simultaneously = 60/11 days.

Approach 2: LCM Method


In this method, we assume the total amount of work to be completed as a finite
divisible value and based on it, we proceed with the calculation. To make the
calculation simpler, assume the total amount of work to be completed as the LCM of
time taken by different people to complete the same piece of work.
Answer:
Let the amount of work be 60 units (LCM of 10 and 12). Since A does 60 units in 10
days, he does 6 units every day. Since B does 60 units in 12 days, he does 5 units
every day. Working simultaneously, they do 6 + 5 = 11 units each day.
Thus to complete 60 units of work, they will take 60/11 days.
The two approaches are absolutely identical, it is just that in the earlier
approach the work was assumed as 1 unit instead of 60 units.
Approach 3: The Chocolate Method
To make calculations more intuitive, the work can be assumed as chocolates to be
consumed rather than units of work to be done. We take LCM of all the "number of
days" mentioned in the question. This is done so that the work will be a multiple
of these "number of days" and thus calculating efficiency will be easier.
For example, if A can do a piece of work in 9 days and B can do the same piece of
work in 18 days, in how many days can they complete it working together?
Answer:
In the chocolate method, we assume work as chocolates.
So, the total amount of chocolate to be consumed = 18 units (LCM of 9 and 18)
This means A can eat 18 chocolates in 9 days.
Chocolate consumed by A in one day = 2 units per day
B, on the other hand, can eat 18 chocolates in 18 days
Chocolate consumed by B in one day = 1 unit per day
Chocolate consumed by A and B in one day = 3 units per day
Time taken by them to consume 18 units of chocolate = 18/3 = 6 days

Basic Concepts of Time and Work

Most of the aptitude questions on time and work can be solved if you know the
basic correlation between time, work and man-hours which you have learnt in your
high school class.


1.
2. Analogy between problems on time and work to time, distance and speed:
1. Speed is equivalent to rate at which work is done
2. Distance travelled is equivalent to work done.
3. Time to travel distance is equivalent to time to do work.
3. Man - Work - Hour Formula:
1. More men can do more work.
2. More work means more time required to do work.
3. More men can do more work in less time.
4. MM men can do a piece of work in TT hours, then Total effort or
work =MT man hoursTotal effort or work =MT man hours.
5. Rate of work * Time = Work DoneRate of work * Time = Work Done
6. If AA can do a piece of work in DD days, then AA's 1 day's work
= 1D1D.
Part of work done by AA for tt days = tDtD.
7. If AA's 1 day's work = 1D1D, then AA can finish the work in DD days.
8.

1.
MDH/W=Constant MDH/W=Constant
Where,

M = Number of men
D = Number of days
H = Number of hours per day
W = Amount of work

2. If M1M1 men can do W1W1 work in D1D1 days working H1H1 hours per day
and M2M2 men can do W2W2 work in D2D2 days working H2H2 hours per day, then

M1D1H1/W1=M2D2H2/W2
Problems on Time and Work : Question 1 :

A can do a certain work in the same time in which B and C together can do it. If A
and B together could do it in 10 days and C alone in 50 days, then B alone could
do it in how many days?

Explanation:
(A + B)’s one day’s work = 1/10. --- (i)
C’s one day’s work = 1/50
(A + B + C)’s one day’s work = (1/10 + 1/50) = 6/50 = 3/25. ---- (ii).
Also, A’s one day’s work = (B + C)’s one day’s work.
From (i) and (ii), we get :2*(A’s one day’s work) = 3/25.
=> A’s one day’s work = 3/50
B’s one day’s work = (1/10 - 3/50)
= 2/50
Thus, B alone could complete the work in 25 days.

Problems on Time and Work : Question 2 :


Pipe A and B can fill a tank in 8 hours, B and C can fill it in 12 hours. A, B and
C together can fill it in 6 days. A and C together will fill it in how many days?

➔Explanation:
Let the total units of work be 48.

Units of work completed by A and B in one day = 6.


Units of work completed by B and C in one day = 4
Units of work completed by A, B, and C in one day = 8.
From the above information we get the work completed by A and C in one day is 6
units.
Therefore, the number of days taken by A and C to complete the whole work = 48/6 =
8 days.

Problems on Time and Work : Question 3 :


Ajay can complete a piece of work in 4 days. Balu takes double the time taken by
Ajay, Charan takes double that of Balu, and Danush takes double that of Charan to
complete the same task. They are paired in groups of two each. One pair takes two
thirds the time needed by the second pair-to complete the work. Which is the first
pair?

➔Explanation:
Time taken by Ajay, Balu, Charan, Dhanush is 4 days, 8 days, 16 days and 32 days
respectively.

Therefore, grouping Ajay and Dhanush work will be two third of the work that will
be completed by grouping Balu and Charan.

Problems on Time and Work : Question 4 :


5 persons working eight hours daily can complete a wall in 10 days. When they have
Worked for 4 days, 5 more persons are brought to work. The wall can now be
completed in?
➔Explanation:
Let D be the number of required days
Using the chain rule,
M1 x T1 = M2 x T2 + M3 x T3

5 x 8 x 10 = (4 x 5 x 8) + (10 x 8 x D)
D = 3.
Hence, the wall will be completed in 3 more days.

Problems on Time and Work : Question 5 :


24 men working at 8 hours a day can finish a work in 10 days. Working at the rate
of 10 hours a day, the number of men required to finish the same work in 6 days is
Explanation:
Using Using the chain rule,
M1 x T1 = M2 x T2
Therefore, 24 x 8 x 10 = 10 x X x 6
X= 32
Hence, the number of men required to finish the work = 32.

Chain rule Principle


After the comparison, the following two principles are followed:

If the missing part is greater than the given part, then the numerator (n) is kept
greater than the denominator (d) i.e. n/d>1, where n & d are the given parts of
other element.

If the missing part is smaller than the given part, then the numerator (n) is kept
smaller than the denominator (d) i.e. n/d<1, where n & d are the given parts of
other elemen

Please study the following illustration to understand this concept.

Illustration 1: 12 examiners (men) work 16 hours a day to check 24000 answer


sheets in 18 days. Now, 24 examiners would work how many hours per day to check
36000 answer sheets in 36 days?

Answer & Explanation

Answer :

Examiners hours answer sheets days

12 16 24000 18

24 ? 36000 36

As we have to calculate the hours in this case, the base would be hours.
Comparisons:

Element = Examiners: If there are 24 examiners now and there were 12 earlier, they
need to work lesser hours per day.

Element = Answer sheets: If there are more answer sheets to be checked now, they
need to work more hours per day.

Element = Days: If there are more days available, then they need to work lesser
hours per day.
Applying chain rule: 16 × (12/24) × (36000/24000) × (18/36) = 6 hours.
In the same illustration if hours were given and answer sheets were missing, then
also the method would have been same. Let us solve the same illustration in that
manner as well.

Illustration 2: 12 examiners work 16 hours a day to check 24000 answer sheets in


18 days. Now, 24 examiners would check how many answer sheets working 6 hours a
day in 36 days?

Answer & Explanation

Answer

Examiners hours answer sheets days

12 16 24000 18

24 6 ? 36

Following the same steps :


As we have to calculate the answer sheets in this case, the base would be answer
sheets.
If there are 24 examiners now and there were 12 earlier, they will check more
answer sheets.
If they work for lesser hours per day, they will check lesser answer sheets.
If there are more days available, then they will check more answer sheets.
24000 × (24/12) × (6/16) × (36/18) = 36000.

Illustration 3: A certain number of men can complete a piece of work in 180 days.
If there are 30 men less, it will take 20 days more for the work to be completed.
How many men were there originally?

1. 135

2. 165

3. 150

4. 180

5. 300

Answer & Explanation

Answer :
Let there be x men originally.
They were to complete the work in 180 days but as the number of persons is reduced
to x – 30.
∴ Work takes 20 more days. So the equation is 180x = (x – 30)200 ⇒ 20x = 6000 ⇒ x
= 300.

Illustration 4: A garrison is provided with ration for 90 soldiers to last for 70


days. For how much more time would the whole ration last if 10 additional soldiers
join them after 20 days?
1. 40 days

2. 36 days

3. 30 days

4. 56¼ days

5. 45 days

Answer & Explanation

Answer :
Let the whole ration now lasts for x days. Equating the consumption on both sides,
we get (90 × 70) = (90 × 20) + (100 × x) ⇒ x = 45 days.

Chain Rule

Points to Remember

1) Direct Proportion: Any two quantities are said to be directly


proportional, if on the increase of one quantity, the other quantity
increases and vice-versa.

Example: Cost is directly proportional to number of objects

Cost ∝ Number of objects


Number of objects increases (↑) Cost (↑)

Example: Work done is directly proportional to number of working men

Work done ∝ Number of working men


Number of men increase (↑) Work done (↑)

2) Indirect Proportion: Any two quantities are said to be indirectly


proportional, if on the increase of one quantity, the other quantity
decreases and vice-versa.

Example: If speed of car is increases, then the time required to cover the
distance decreases.
Speed of car (↑) Time required decreases (↓)

Example: Time taken to finish work increases, if number of men decrease.


Time (↑) Number of men (↓)

Tips and Tricks

In this chapter generally different types of numerical related to time and


work, time and speed, cost and number of articles, men and work, etc. are
asked.

Type 1: Indirect proportion


Q 1. If 30 men can do a piece of work in 20 hours, then in how many hours
will 12 men do it?
Hint: As number of workers increase, the time required decreases. Hence,
this is a problem related to indirect proportion.
Workers (↑),Time (↓)
Let the number of hours be x.
12 : 30 :: 20 : x

20 20
=
30 x

20 × 30
x = = 50
12

12 men require 50 hours to complete the same work.

Q 2. 3 pumps, working 4 hours a day, can empty a tank in 2 days. How many
hours a day must 4 pumps work, to empty the tank in one day?
Hint: As number of pumps increase, the time required decreases and when
working hours increase, fewer days are required to complete the work.
Hence, this is a problem related to indirect proportion.
Given: 3 pumps can empty a tank in 2 days, if they are working 6 hours a
day
Find: Number of hours a day, 3 pumps must work, to empty the tank in one
day.
More pumps (↑),Less working hours (↓)
More working hours (↑),Less days (↓)

Pumps are in the ratio 4 : 3


4 : x ::
Days are in the ratio 1 : 2

4 × 3 × 2 = 4 × 1 × x
24 = 4x
x = 6

Alternate solution 1: (Trick)


Arrange all given parameters in table format.

Pumps Days Hours

3 2 4

4 1 ’A’ hrs

Simply multiply, we get


3 × 2 × 4 = 4 × 1 × A

3 × 2 × 4
A = = 6 hrs
4 × 1

Alternate solution 2:
We are given that, 3 pumps, working 4 hours a day, can empty a tank in 2
days. Therefore, it means that:
3 pumps take total 8 hours to empty the tank.
Hence, 1 pump will take 8 x 3 = 24 hours
Remember: As number of pump decrease, time required increases.
So, if 4 pumps work, time required decreases.

24
= 6 hrs
4

Q 3. A wheel that has 6 cogs is meshed with a larger wheel of 12 cogs. If


the smaller wheel has made 22 revolutions, then find the number of
revolutions made by the larger wheel.
Hint: As number of cogs increase, the revolutions made decrease. Hence,
this is a problem related to indirect proportion.
Let the number of wheels be x.
More cogs (↑),Less revolutions (↓)
Given: 6 cogs meshed with wheel of 12 cogs and smaller wheel made 22
revolutions
Therefore,
12:6 :: 22:x
12 × x=6 × 22

6 × 22
x = = 11
12

Type 2: Direct proportion

Q 4. If 30 men can build a wall 56 meters long in 5 days, what length of a


similar wall can be built by 40 men in 3 days?

Hint: If more men work, length of wall built is more. If worked for few
days, the length of wall built is also less. Hence, this problem is related
to direct proportion.
The two main parameters are man and days.
Therefore,

30 : 40 - - - (Men)
56 : x ::
5 : 3 - - - (Days)
30 × 5 × x = 40 × 3 × 56

40 × 3 × 56
x = = 44.8
30 × 5

Q 5. 5 mat-weavers can weave 5 mats in 5 days. At the same time, how many
mats would be woven by 10 mat- weavers in 10 days?

Hint: More mats are weaved if more weavers work. Hence, this problem is
related to direct proportion.
Let the required number of mats be x.
Total 5 mats can be weaved in 5 days by 5 weavers.

5 : 10 - - - (Weavers)
5 : x ::
5 : 10 - - - (Days)

5 × 5 × x = 10 × 10 × 5

10 × 10 × 5
x = = 20
5 × 5

20 mats can be weaved in 10 days by 10 mat weavers.

Q 6. If 8 men can reap 40 hectares in 12 days, then how many hectares can
30 men reap in 20 days?

Let the required number of hectares be x.


More men (↑) More hectares (↑)

8 : 30 - - - (Men)
40 : x ::
12 : 20 - - - (Hectares)

8 × 12 × x = 30 × 20 × 40

30 × 20 × 40
x = = 250 Hectares
8 × 12

Type 3: Combination of Direct and Indirect proportion

Q 8. In a diary farm, 20 cows eat 20 bags in 20 days. In how many days one
cow will eat one bag of husk?

Indirect proportion: Less cows (↓) More days (↑)


Direct proportion: Less bags (↓) Less days (↓)
1 : 20 - - - (Cows)
20 : x ::
20 : 1 - - - (Bags)

1 × 20 × x = 20 × 1 × 20

20 × 20 × 1
x = = 20 Days
20 × 1

Q 9. 18 men bind 900 books in 10 days. Find how many binders will be
required to bind 600 books in 12 days?
we have to find the number of binders. Let the number of binders be x.
Direct Proportion:Less Books (↓),Less binders(↓)
Indirect Proportion:More days (↑),Less binders (↓)

900 : 600 - - - (Books)


18 : x ::
12 : 10 - - - (Days)

x × 900 × 12 = 18 × 600 × 10

18 × 600 × 10
x = = 10
900 × 12

Unit7 Time speed and distance


Basic

In this article, we will discuss the concepts, formulas and some questions based
on speed, time and distance. As Time, Speed and Distance forms an integral part of
every competitive exam, you cannot afford to skip this topic.

Speed

Speed basically tells us how fast or slow an object moves.


It is described as the distance travelled by an object divided with the time taken
to cover that distance.

Speed = Distance/Time

This shows that Speed is directly proportional to distance but inversely


proportional to time.

Distance = Speed * Time and,

Time = Distance/Speed

Example: What is the distance covered by a car travelling at a speed of 40 kmph in


15 minutes?

Solution:

Distance= speed* time= 40*15/60=10 km.

The important point to note is that time given was in minutes, whereas the speed
was in kmph.

Therefore, either speed has to be converted to km/min or time has to be expressed


in hours.

We have converted time in hours.

15 min=15/60 hours.

Average Speed

Case 1: When Time is Constant

The average speed of travelling at two different speeds for the same time span is
just the simple average of two speeds.

Let Speed 1 be x km/hr

Let Speed 2 be y km/hr

Therefore,

Average Speed when time is same = (x+y)/2

Example: A car is travelling at an average speed of 45kmph for the 1st hour and at
65 kmph for the next 1 hour. Calculate his average speed.

Solution:
As the time is same, i.e. 1 hour,
Average speed= (45+65)/2= 55 kmph.

Must Read Time, Speed and Distance Articles

Case 2: Average Speed When Distance is Constant

Average Speed = 2ab/(a+b) (where a and b are two speeds)


Let us understand how this came.

Let the two speeds be a km/hr and b km/hr.

Let the distance travelled in each of the speeds be x km.

As we know that, Time = Distance/Speed

Hence, time taken to cover x km at a km/hr will be x/a hrs

And, time taken to cover x km at b km/hr will be x/b hrs

Total time taken = x/a+x/b =(bx+ax)/ab = x(b+a)/ab

And the total distance covered = 2x

Therefore,

Average Speed =

Example: On his way to office, Big Bull was travelling at 30 kmph and on the
return journey, he was travelling at 45kmph. What is Big Bull's average speed?

Solution:

37.5 kmph is incorrect as the time travelled is different in both the cases and
only the distances are same.

Let distance = x km

Therefore, Time taken on Big Bull's onward journey =x/30 hours and

Time taken on his return journey=x/45 hours

Therefore, total time = (x/30)+ (x/45) hours.

Total distance = 2x km

Average speed= kmph = 36kmph

Important Time and Distance Conversions

1 km = 1000 meters

1 meter = 100 cm

1 hour = 60 minutes

1 min = 60 seconds

1 hours = 3600 seconds

1 km/hr = m/sec

Hence, 1 m/sec = km/hr


1 mile = 1760 yards

1 yard = 3 feet

1 mile = 5280 feet

1 mph = yards/sec

1 mph = ft/sec

Now, let us try doing some questions.

Solved Questions

Question 1: What is the distance covered by a car travelling at a speed of 40 kmph


in 15 minutes?

Solution:

We know that 1 hour = 60 minutes

Therefore, 15 minutes = 1/4 hours (because 15/60 = 1/4)

Distance = Speed * Time

Distance = 40 * (1/4) = 10 kms

Question 2: A car is travelling at a speed of 50 kmph. How long will it take to


travel 60 kms?

Solution: Distance = Speed * Time

60 = 50 * T

T = 1.2 hrs or 1 hr and 12 minutes

Question 3: Walking at 5/6th of his usual seed, Raman reached his destination 10
minutes late. Find his usual time and he times taken on this occasion.

Solution:

Let his usual speed be x km/hr and his usual time be t hours

His time on this occasion is

The time taken is hrs

Since the distance travelled on both occasions is the same,

xt = * ( because, Distance = Speed * Time)

Solving for t, we get t = 5/6hrs


= 50 minutes and the time taken on this occasion = 50 + 10 = 60 minutes

Question 4: While going to office, Rajat travels at a speed of 30 kmph and on his
way back, he travels at a speed of 45 kmph. What is his average speed of the whole
journey?

Solution: When distance travelled is same, then average speed = 2ab/(a+b)(where a


and b are two different speeds)

Therefore, Average Speed =

Average Speed = 36 kmph

Question 5: A CTU bus and an ordinary bus leave Chandigarh for Ambala- a distance
of 32 km simultaneously. The ratio between the average speed of the CTU bus and
that of the ordinary bus is 3:2. The CTU bus reaches Ambala and immediately leaves
back for Chandigarh and meets the ordinary bust Lalru. What is the distance
between Ambala and Lalru?

Solution: Speed of CTU bus : Speed of ordinary bus :: 3:2

As Distance ∞ Speed

Distance travelled by CTU bus : Distance travelled by ordinary bus :: 3:2

Let the distance travelled between Ambala and Lalru be x km

Then the distance travelled by CTU bus = 32 + x

While the distance travelled by ordinary bus = 32 – x

Therefore, 32 + x : 32 – x :: 3:2

Solving for x, we get x = 6.4 km (i.e. the distance between Ambala and Lalru)

Question 6: Travelling at 6 kmph, Niharika reaches her office 20 minutes late.


Travelling at 8 kmph, she reaches her office 30 minutes early. What is her usual
speed and time taken to reach her office?

Solution: Let her usual speed be S kmph and her usual time be t hours

Therefore,

Solving for t, we get t = 3 hrs

Since the usual time taken = 3 hrs, usual distance travelled = 3S kms

Equating distance travelled usually, with distance travelled at any of the other
two speeds, we get 6 * (3+20/60) = 3S

Therefore, S = 6 kmph

Question 7: How much time will a 360 meters long train running at 108 kmph will
take to cross a standing electricity pole?
Solution: We know that, 1 km/hr = 5/18 m/sec

Hence, 108 kmph = 108 * m/sec = 30 m/sec

Distance = Speed * Time

360 = 30 * Time

Time = 12 secs

Time Speed Distance


Time, Speed and Distance (popularly known as TSD) is an important topic for
written round of placements for any company.
• Distance = Speed x Time
• To convert from km / hour to m / sec, we multiply by 5 / 18. So, 1 km / hour =
5 / 18 m / sec
• To convert from m / sec to km / hour, we multiply by 18 / 5. So, 1 m / sec = 18
/ 5 km / hour = 3.6 km / hour
• For a certain distance, if the ratio of speeds is x : y, then the ratio of
times taken to cover the distance would be y : x and vice versa.
• If a person covers a certain same distance multiple times with different
speeds, then
Average speed = n / ∑ (1/si), where n is the number of times the distance is
covered and si are the respective speeds of covering the distance.
For example, if a person travels a distance of 10 km three times at the speeds
of 4 km/h, 5 km/h and 6 km/h, then the average speed would be 3 / [ (1 / 4) +
(1 / 5) + (1 / 6) ] = 3 / (37 / 60) = 180 / 37 &approx; 4.86 km/h
• Relative Speed
• If two people / objects are moving in same direction with speeds x km / h
and y km / h (x > y), then their relative speed would be (x – y) km / h
• If two people / objects are moving in opposite direction with speeds x km /
h and y km / h, then their relative speed would be (x + y) km / h
• Relative speed is the rate at which two moving bodies are separating from /
coming closer to each other. For example, if two persons are moving at 10
km/h and 20 km/h in opposite directions, then their relative speed would be
10 + 20 = 30 km / h, i.e., the distance between them after one hour would
be 30 km. Similarly, if they were moving in the same direction, their
relative speed would be 20 – 10 = 10 km / h, i.e., the distance between
them after one hour would be 10 km.

Sample Problems

Question 1 : A runner can complete a 750 m race in two and a half minutes.
Will he be able to beat another runner who runs at 17.95 km / hr ?
Solution : We are given that the first runner can complete a 750 m race in 2
minutes and 30 seconds or 150 seconds.
=> Speed of the first runner = 750 / 150 = 5 m / sec
We convert this speed to km / hr by multiplying it by 18/5.
=> Speed of the first runner = 18 km / hr
Also, we are given that the speed of the second runner is 17.95 km / hr.
Therefore, the first runner can beat the second runner.

Question 2 : A man decided to cover a distance of 6 km in 84 minutes. He decided


to cover two thirds of the distance at 4 km / hr and the remaining at some
different speed. Find the speed after the two third distance has been covered.
Solution : We are given that two thirds of the 6 km was covered at 4 km / hr.
=> 4 km distance was covered at 4 km / hr.
=> Time taken to cover 4 km = 4 km / 4 km / hr = 1 hr = 60 minutes
=> Time left = 84 – 60 = 24 minutes
Now, the man has to cover remaining 2 km in 24 minutes or 24 / 60 = 0.4 hours
=> Speed required for remaining 2 km = 2 km / 0.4 hr = 5 km / hr

Question 3 : A postman traveled from his post office to a village in order to


distribute mails. He started on his bicycle from the post office at the speed of
25 km / hr. But, when he was about to return, a thief stole his bicycle. As a
result, he had to walk back to the post office on foot at the speed of 4 km / hr.
If the traveling part of his day lasted for 2 hours and 54 minutes, find the
distance between the post office and the village.
Solution : Let time taken by postman to travel from post office to village=t
minutes.
According to the given situation, distance from post office to village, say
d1=25/60*t km {25 km/hr = 25/60 km/minutes}
And
distance from village to post office, say d2=4/60*(174-t) km {2 hours 54 minutes =
174 minutes}
Since distance between village and post office will always remain same i.e. d1 =
d2
=> 25/60*t = 4/60*(174-t) => t = 24 minutes.
=> Distance between post office and village = speed*time =>25/60*24 = 10km

Question 4 : Walking at the speed of 5 km / hr from his home, a geek misses his
train by 7 minutes. Had he walked 1 km / hr faster, he would have reached the
station 5 minutes before the actual departure time of the train. Find the distance
between his home and the station.
Solution : Let the distance between his home and the station be ‘d’ km.
=> Time required to reach the station at 5 km / hr = d/5 hours
=> Time required to reach the station at 6 km / hr = d/6 hours
Now, the difference between these times is 12 minutes = 0.2 hours. (7 minutes late
– 5 minutes early = (7) – (-5) = 12 minutes)
Therefore, (d / 5) – (d / 6) = 0.2
=> d / 30 = 0.2
=> d = 6
Thus, the distance between his home and the station is 6 km.

Question 5 : Two stations B and M are 465 km distant. A train starts from B
towards M at 10 AM with the speed 65 km / hr. Another train leaves from M towards
B at 11 AM with the speed 35 km / hr. Find the time when both the trains meet.

Solution : The train leaving from B leaves an hour early than the train that
leaves from M.
=> Distance covered by train leaving from B = 65 km / hr x 1 hr = 65 km
Distance left = 465 – 65 = 400 km
Now, the train from M also gets moving and both are moving towards each other.
Applying the formula for relative speed,
Relative speed = 65 + 35 = 100 km / hr
=> Time required by the trains to meet = 400 km / 100 km / hr = 4 hours
Thus, the trains meet at 4 hours after 11 AM, i.e., 3 PM.

Question 6 : A policeman sighted a robber from a distance of 300 m. The robber


also noticed the policeman and started running at 8 km / hr. The policeman also
started running after him at the speed of 10 km / hr. Find the distance that the
robber would run before being caught.
Solution : Since both are running in the same direction, relative speed = 10 – 8 =
2 km / hr
Now, to catch the robber if he were stagnant, the policeman would have to run 300
m. But since both are moving, the policeman needs to finish off this separation of
300 m.
=> 300 m (or 0.3 km)is to be covered at the relative speed of 2 km / hr.
=> Time taken = 0.3 / 2 = 0.15 hours
Therefore, distance run by robber before being caught = Distance run in 0.15 hours
=> Distance run by the robber = 8 x 0.15 = 1.2 km

Another Solution :
Time of running for both the policeman and the robber is same.
We know that Distance = Speed x Time
=> Time = Distance / Speed
Let the distance run by the robber be ‘x’ km at the speed of 8 km / hr.
=> Distance run by policeman at the speed of 10 km / hr = x + 0.3
Therefore, x / 8 = (x + 0.3) / 10
=> 10 x = 8 (x + 0.3)
=> 10 x = 8 x + 2.4
=> 2 x = 2.4
=> x = 1.2
Therefore, Distance run by the robber before getting caught = 1.2 km

Problem on Time Speed and Distance

Question 1: A racing car covers a certain distance at a speed of 320 km/hr in 5


hours. To cover the same distance in 5/3 hours it must travel at a speed of:
Solution: Given Distance is constant.
So, Speed is inversely proportional to time.
Ratio of time 5 : 5/3
Ratio of time becomes 3 : 1
Then, Ratio of speed 1 : 3
1 unit -> 320 km/hr
3 unit -> 320 x 3 = 960 km/hr is required speed

Question 2: A train running at a speed of 36 km/hr and 100 meter long. Find the
time in which it passes a man standing near the railway line is :
Solution: Speed = 36 km/hr
Change in m/s
So, speed = 36 * 5/18 = 10 m/s
Time required = Distance/speed
= 100/10
= 10 second

Question 3: If an employee walks 10 km at a speed of 3 km/hr, he will be late by


20 minutes. If he walks at 4 km/hr, how early from the fixed time he will reach ?
Solution: Time taken at 3 km/hr = Distance/speed
= 10/3
Actual time is obtained by subtracting the late time
So, Actual time = 10/3 – 1/3 = 9/3 = 3 hour
Time taken at 4 km/hr = 10/4 hr
Time difference = Actual time – time taken at 4 km/hr
= 3 – 10/4
= 1/2 hour
Hence, he will be early by 30 minutes.
Question 4: The diameter of each wheel of a truck is 140 cm, If each wheel rotates
300 times per minute then the speed of the truck (in km/hr) (take pi=22/7)
Solution: Circumference of the wheel= 2 * 22/7 * r
= 2 * 22/7 * 140/2
= 440 cm
Speed of the car = (440 * 300 * 60)/(1000 * 100 )
= 79.2 km/hr
Question 5: A man drives at the rate of 18 km/hr, but stops at red light for 6
minutes at the end of every 7 km. The time that he will take to cover a distance
of 90 km is

Solution: Total Red light at the end of 90 km = 90/7 = 12 Red light + 6 km


Time taken in 12 stops= 12 x 6 = 72 minutes
Time taken by the man to cover the 90 km with 18 km/hr without stops = 90/18 = 5
hours
Total time to cover total distance = 5 hour + 1 hour 12 minute
= 6 hour 12 minute

Question 6: Two jeep start from a police station with a speed of 20 km/hr at
intervals of 10 minutes.A man coming from opposite direction towards the police
station meets the jeep at an interval of 8 minutes.Find the speed of the man.
Solution:
Jeep Jeep + man
Ratio of time 10 min : 8 min
Ratio of speed 8 : 10
4 : 4+1
Here, 4 units -> 20 km/hr
1 unit -> 5 km/hr
Speed of the man = 1 unit = 1 x 5 = 5 km/hr

Question 7: Two city A and B are 27 km away. Two buses start from A and B in the
same direction with speed 18 km/hr and 24 km/hr respectively. Both meet at point C
beyond B. Find the distance BC.

Solution: Relative speed = 24 – 8


= 6 km/hr
Time required by faster bus to overtake the slower bus = Distance/time
=27/6 hr
Distance between B and C= 18*(27/6)= 81 km

Question 8: A man travels 800 km by train at 80 km/hr, 420 km by car at 60 km/hr


and 200 km by cycle at 20 km/hr. What is the average speed of the journey?
Solution: Avg. Speed = Total distance/time taken
(800 + 420 + 200) / [(800/80) + (420/60) + (200/20)]
=>1420 / (10 + 7 + 10)
=>1420/27
=>1420/27 km/hr

Question 9: Ram and Shyam start at the same with speed 10 km/hr and 11 km/hr
respectively.
If Ram takes 48 minutes longer than Shyam in covering the journey, then find the
total distance of the journey.
Solution:
Speed Ratio 10 : 11
Time ratio 11 : 10
Ram takes 1 hour means 60 minutes more than Shyam.
But actual more time = 48 minute.
60 unit -> 48 min
1 unit -> 4/5
Distance travelled by them= Speed x time
= 11 x 10 = 110 unit
Actual distance travelled = 110 x 4/5
= 88 km

Question 10: A person covered a certain distance at some speed. Had he moved 4
km/hr faster, he would have taken 30 minutes less. If he had moved 3 km/hr slower,
he would have taken 20 minutes more. Find the distance (in km)
Solution: Distance = [S1S2/ (S1– S2)] x T
S1 = initial speed
S2 = new speed
Distance travelled by both are same so put equal
[S (S + 4) / 4 ] * (30/60) =[ S (S – 3)/ 3 ]* (30/60)
S = 24
Put in 1st
Distance=(24 * 28) / 4 * (30/60) = 84 km

Question 11: Ram and Shyam start from the same place P at same time towards Q.
Ram’s speed is 4 km/hr more than that of Shyam.Ram turns back after reaching Q and
meet Shyam at 12 km distance from Q.Find the speed of Shyam.
Solution: Let the speed of the Shyam = x km/hr
Then Ram speed will be = (x + 4) km/hr
Total distance covered by Ram = 60 + 12 = 72 km
Total distance covered by Shyam = 60 – 12 = 48 km
Acc. to question, their run time are same.
72/ (x + 4) = 48/ x
72x = 48x + 192
24x= 192
x= 8
Shyam speed is 8 km/hr

Question 12: A and B run a kilometre and A wins by 20 second. A and C run a
kilometre and A wins by 250 m. When B and C run the same distance, B wins by 25
second. The time taken by A to run a kilometre is
Solution: Let the time taken by A to cover 1 km = x sec
Time taken by B and C to cover the same distance are x + 20 and x + 45
respectively
Given A travels 1000 then C covers only 750.

Distance A(1000) C(750)


Ratio 4 : 3
Time 3 : 4
A/C = 3/4 = x/(x+45)
3x + 135 = 4x
x =135
Time taken by A is 2 min 15 second
Problem on Trains, Boat and streams
Question 1: A train passes two bridges of length 1000 m and 600 m in 120 seconds
and 80 seconds respectively. The length of the train.

Solution: Distance covered in 120 second = 1000 + length of train(l)


Distance covered in 80 seconds = 600 + l
So, distance covered in 40 seconds = (1000 + l) – (600 + l)
= 400 m
Speed = 400/40 = 10 m/s
Distance covered in 80 second = 80 x 10 = 800 m
So, 600 + l = 800
Length of the train (l) = 200 m
Question 2: A train 500 m long is running at a speed of 72 km/hr. If it passes
through a tunnel in 50 seconds, then the length of the tunnel is :

Solution: First convert speed in m/s


So, speed= 72 x (5/18)
= 20 m/s
Train covers the distance in 50 seconds = length of train + length of the
tunnel(l)
500 + l = 20 x 50
500 + l = 1000
l = 500 m

Question 3: A train reaches from A to B in 5 hours travelling at a speed of 60


km/hr. If its speed is increased by 15 km/hr, then the time of journey is reduced
by.

Solution: Total distance = speed x time


=60 x 5 = 300 km
If speed increased then new speed= 60 + 15 = 75 km/hr
New time = Total distance/speed
= 300/75= 4 hour
Time reduced by 5 – 4 = 1 hour

Question 4: Delhi and Mumbai apart from each other 760 km.A train starts from
Delhi at 9 am and travels towards Mumbai at speed 60 km/hr. Another train starts
from Mumbai at 10 am and travels towards Delhi at speed 80 km/hr. At what time
both will meet?
Solution: Total distance between D and M = 760 km.
A travels 1 hour before B so it travels = 60 x 1 = 60 km
Now the remaining distance D and M= 760 – 60 = 700 km
Relative speed = 60 + 80 = 140 km/hr
Time = 700 / 140
= 5 hour.
So, the time when they meet = 10 am + 5 hour = 3 pm

Question 5: Two trains 180 m and 120 m long respectively pass each other in 54
seconds when they run in the same direction and in 18 seconds when run in opposite
directions. Find the speed of two trains.
Solution: Let the speed of 1st train is S1 and speed of 2nd train is S2
Time = total distance/ relative speed
1) In same direction
54 = (180 + 120) / (S1 – S2) * 5/18
(S1 – S2)54 = (300 * 18)/5
(S1 – S2) = 20
2) In opposite direction
9 = (180 + 120) / (S1 + S2) * 5/18
(S1 + S2)18 = (300 * 18)/5
(S1 + S2) = 60
from 1 and 2
S1 = 40 km/hr
S2 = 20 km/hr

Question 6: Two trains start from station A and B and travels towards each other
at speed of 48km/hr and 72km/hr respectively. At the time of their meeting, the
second train has traveled 144 km more than the first. The distance between A and B
is:

Solution: The second train has traveled 144 km more than the first train because
the speed of second train is 24 km/hr more than first.
Time taken by second train to cover 144 km with surplus 24km/hr = 144/24 = 6
hours.
then, time taken by both train before meeting is 6 hours.
So, their relative speed = 48 + 72 = 120
Total distance travel by both = 120 x 6 = 720 km
Distance between A and B = 720 km

Question 7: If the speed of the boat in still water is 5 km/hr and the speed of
the current is 10 km/hr, then find the time taken by the boat to travel 125 km
with the current.
Solution: Relative speed = 15 + 10
=25 km/hr
Time = Distance/speed
= 125/25
= 5 hour

Question 8: On a river, C is the mid-point between two points A and B on the same
bank of the river. A boat can go from A to C and back in 14 hours and from A to B
in 20 hours 20 min. How long it would take to go from B to A?
Solution: Time required to travel from A to B = 20 hour 20 min
Time required to travel from A to C = 1/2 (20 h 20 m)
= 10 h 10 m
Given total time from A to C and C to A = 14 h
10 h 10 m + C to A = 14 h
C to A = 3 h 50 m
Time taken from B to A is twice of C to A
then, time taken from B to A =2*(3 h 50 m)=7 h 40 m

Question 9: The ratio of speed of a motor-boat to that of the current of water is


17 : 5. The boat goes along with the current in 4 hours. It will come back in
Solution: Since the ratio 17 : 5 is given.
Let the speed of boat in still water = 17 km/hr and speed of stream = 5 km/hr
Downstream speed = 17 + 5 = 22 km/hr
Upstream speed = 17 – 5 = 12 km/hr
Distance = Downstream speed x downstream time
= 22 x 4 = 88 km
Upstream time = Distance/upstream speed
= 88/12
Come back time = 7 hour 20 minute

Question 10: Speed of motorboat in still water is 35 kmph. If the motorboat


travels 100 km along the stream in 2 hour 30 min, then the time taken by it to
cover the same distance against the stream is
Solution: The speed of the motorboat in still water is 35 km/hr.
let the speed of the strem = x km/hr
Downstream speed = Distance/time
= 100 / 2.5
= 40 km/hr
Speed of stream = 35 + x = 40
x = 5 km/hr
Upstream speed = 35 – 5 = 30 km/hr
Time taken in upstream = 100/30 = 3 hour 20 mi
Unit 8 Permutation and combination probability
Basic
Important Concepts and Formulas - Permutations and Combinations1. Multiplication
Theorem (Fundamental Principles of Counting)
If an operation can be performed in mm different ways and following which a second
operation can be performed in nn different ways, then the two operations in
succession can be performed in m×nm×n different ways.

2. Addition Theorem (Fundamental Principles of Counting)

If an operation can be performed in mm different ways and a second independent


operation can be performed in nn different ways, either of the two operations can
be performed in (m+n)(m+n) ways.

3. Factorial

Let nn be a positive integer. Then nn factorial can be defined as


n!=n(n−1)(n−2)⋯1n!=n(n−1)(n−2)⋯1

Examples

5!=5×4×3×2×1=120 3!=3×2×1=65!=5×4×3×2×1=120 3!=3×2×1=6

Special Cases

0!=1 1!=10!=1 1!=1

4. Permutations
Permutations are the different arrangements of a given number of things by taking
some or all at a time.

Examples

All permutations (or arrangements) that can be formed with the letters a, b, c by
taking three at a time are (abc, acb, bac, bca, cab, cba)

All permutations (or arrangements) that can be formed with the letters a, b, c by
taking two at a time are (ab, ac, ba, bc, ca, cb)

5. Combinations
Each of the different groups or selections formed by taking some or all of a
number of objects is called a combination.

Examples

Suppose we want to select two out of three girls P, Q, R. Then, possible


combinations are PQ, QR and RP. (Note that PQ and QP represent the same
selection.)

Suppose we want to select three out of three girls P, Q, R. Then, only possible
combination is PQR
6. Difference between Permutations and Combinations and How to identify them

Sometimes, it will be clearly stated in the problem itself whether permutation or


combination is to be used. However if it is not mentioned in the problem, we have
to find out whether the question is related to permutation or combination.

Consider a situation where we need to find out the total number of possible
samples of two objects which can be taken from three objects P, Q, R. To
understand if the question is related to permutation or combination, we need to
find out if the order is important or not.

If order is important, PQ will be different from QP, PR will be different from RP


and QR will be different from RQ

If order is not important, PQ will be same as QP, PR will be same as RP and QR


will be same as RQ

Hence,
If the order is important, problem will be related to permutations.
If the order is not important, problem will be related to combinations.

For permutations, the problems can be like "What is the number of permutations the
can be made", "What is the number of arrangements that can be made", "What are the
different number of ways in which something can be arranged", etc.

For combinations, the problems can be like "What is the number of combinations the
can be made", "What is the number of selections the can be made", "What are the
different number of ways in which something can be selected", etc.

pq and qp are two different permutations, but they represent the same combination.

Mostly problems related to word formation, number formation etc will be related to
permutations. Similarly most problems related to selection of persons, formation
of geometrical figures, distribution of items (there are exceptions for this) etc
will be related to combinations.

7. Repetition

The term repetition is very important in permutations and combinations. Consider


the same situation described above where we need to find out the total number of
possible samples of two objects which can be taken from three objects P, Q, R.

If repetition is allowed, the same object can be taken more than once to make a
sample. i.e., PP, QQ, RR can also be considered as possible samples.

If repetition is not allowed, then PP, QQ, RR cannot be considered as possible


samples.

Normally repetition is not allowed unless mentioned specifically.

8. Number of permutations of n distinct things taking r at a time

Number of permutations of n distinct things taking r at a time can be given by


nP = n!(n−r)!n!(n−r)! =n(n−1)(n−2)...(n−r+1)=n(n−1)(n−2)...(n−r+1) where 0≤r≤n0≤r≤
r
n
Special Cases
n
P0 = 1
nP
r = 0 for r>nr>n

n
Pr is also denoted by P(n,r). nPr has importance outside combinatorics as well
where it is known as the falling factorial and denoted by (n)r or nr

Examples

8
P2 = 8 × 7 = 56
5P = 5 × 4 × 3 × 2 = 120
4

9. Number of permutations of n distinct things taking all at a time

Number of permutations of n distinct things taking them all at a time


= nPn = n!

10. Number of Combinations of n distinct things taking r at a time

Number of combinations of n distinct things taking r at a time ( nCr) can be given


by
nC
r = n!(r!)(n−r)!n!(r!)(n−r)! =n(n−1)(n−2)⋯(n−r+1)r!=n(n−1)(n−2)⋯(n−r+1)r! where
0≤r≤n0≤r≤n

Special Cases
nC
0 = 1
nC
r = 0 for r>nr>n

nC is also denoted by C(n,r). nCr occurs in many other mathematical contexts as


r
well where it is known as binomial coefficient and denoted by (nr)(nr)

Examples

8C
2 = 8×72×18×72×1 = 28

5C =
4 5×4×3×24×3×2×15×4×3×24×3×2×1 = 5

Here’s an easy way to remember: permutation sounds complicated, doesn’t it? And it is. With permutations, every little detail
matters. Alice, Bob and Charlie is different from Charlie, Bob and Alice (insert your friends’ names here).

Combinations, on the other hand, are pretty easy going. The details don’t matter. Alice, Bob and Charlie is the same as Charlie,
Bob and Alice.

Permutations are for lists (order matters) and combinations are for groups (order doesn’t matter).

A joke: A "combination lock" should really be called a "permutation lock". The order you put the numbers in matters. (A true
"combination lock" would accept both 10-17-23 and 23-17-10 as correct.)

Permutations: The Hairy Details


Let’s start with permutations, or all possible ways of doing something. We’re using the fancy-pants term “permutation”, so
we’re going to care about every last detail, including the order of each item. Let’s say we have 8 people:

1: Alice
2: Bob
3: Charlie
4: David
5: Eve
6: Frank
7: George
8: Horatio
How many ways can we award a 1st, 2nd and 3rd place prize among eight contestants? (Gold / Silver / Bronze)

We’re going to use permutations since the order we hand out these medals matters. Here’s how it breaks down:

• Gold medal: 8 choices: A B C D E F G H (Clever how I made the names match up with letters, eh?). Let’s say A wins
the Gold.
• Silver medal: 7 choices: B C D E F G H. Let’s say B wins the silver.
• Bronze medal: 6 choices: C D E F G H. Let’s say… C wins the bronze.

We picked certain people to win, but the details don’t matter: we had 8 choices at first, then 7, then 6. The total number of
options was 8 · 7 · 6 = 336.

Let’s look at the details. We had to order 3 people out of 8. To do this, we started with all options (8) then took them away one at
a time (7, then 6) until we ran out of medals.

We know the factorial is:

Unfortunately, that does too much! We only want 8 · 7 · 6. How can we “stop” the factorial at 5?

This is where permutations get cool: notice how we want to get rid of 5 · 4 · 3 · 2 · 1. What’s another name for this? 5 factorial!

So, if we do 8!/5! we get:

And why did we use the number 5? Because it was left over after we picked 3 medals from 8. So, a better way to write this
would be:

where 8!/(8-3)! is just a fancy way of saying “Use the first 3 numbers of 8!”. If we have n items total and want to pick k in a
certain order, we get:

And this is the fancy permutation formula: You have n items and want to find the number of ways k items can be ordered:

Combinations, Ho!
Combinations are easy going. Order doesn’t matter. You can mix it up and it looks the same. Let’s say I’m a cheapskate and
can’t afford separate Gold, Silver and Bronze medals. In fact, I can only afford empty tin cans.

How many ways can I give 3 tin cans to 8 people?

Well, in this case, the order we pick people doesn’t matter. If I give a can to Alice, Bob and then Charlie, it’s the same as giving
to Charlie, Alice and then Bob. Either way, they’re equally disappointed.

This raises an interesting point — we’ve got some redundancies here. Alice Bob Charlie = Charlie Bob Alice. For a moment,
let’s just figure out how many ways we can rearrange 3 people.

Well, we have 3 choices for the first person, 2 for the second, and only 1 for the last. So we have 3 · 2 · 1 ways to re-arrange 3
people.

Wait a minute… this is looking a bit like a permutation! You tricked me!

Indeed I did. If you have N people and you want to know how many arrangements there are for all of them, it’s just N factorial
or N!

So, if we have 3 tin cans to give away, there are 3! or 6 variations for every choice we pick. If we want to figure out how many
combinations we have, we just create all the permutations and divide by all the redundancies. In our case, we get 336
permutations (from above), and we divide by the 6 redundancies for each permutation and get 336/6 = 56.

The general formula is

which means “Find all the ways to pick k people from n, and divide by the k! variants”. Writing this out, we get
our combination formula, or the number of ways to combine k items from a set of n:

Sometimes C(n,k) is written as:

which is the the binomial coefficient.

A Few Examples
Here’s a few examples of combinations (order doesn’t matter) from permutations (order matters).

• Combination: Picking a team of 3 people from a group of 10. C(10,3) = 10!/(7! · 3!) = 10 · 9 · 8 / (3 · 2 · 1) = 120.

Permutation: Picking a President, VP and Waterboy from a group of 10. P(10,3) = 10!/7! = 10 · 9 · 8 = 720.

• Combination: Choosing 3 desserts from a menu of 10. C(10,3) = 120.

Permutation: Listing your 3 favorite desserts, in order, from a menu of 10. P(10,3) = 720.

Don’t memorize the formulas, understand why they work. Combinations sound simpler than permutations, and they are.
You have fewer combinations than permutations.

You might also like